· web viewfederal income tax outline. fall ‘13, buchanan. introduction. terminology...

105
FEDERAL INCOME TAX OUTLINE FALL ‘13, BUCHANAN I.INTRODUCTION a. Terminology §7701(a)(1) – “Person” is defined as an individual, trust, estate or corp. Ability to pay : the attribute that might justify requiring some people to pay more tax than others. Average tax rate = total tax/total income Cash-flow consumption tax : simply an income tax with a deduction for savings and with the inclusion in the tax base of amounts drawn down from savings and used for consumption, as well as amounts borrowed for current consumption. Double Taxation – taxing the same basis twice Endowment or wage rate (the rate at which one can earn money): the opportunity to earn wealth whether or not exercised. o Flat tax is generally not proportional and it’s appeal has more to do with simplification and ease of administration than fairness Flow variable – defined with respect to passage of time (e.g. income as it goes up and down) Flow variable : variable whose value is determined with reference to the passage of time Global approach : income from all sources is combined and taxed under a rate schedule that applies to the taxpayer's entire income, net any allowable losses or deductions Imputed income : the value of goods and services provided to oneself. Incidence – the ultimate burden of tax, who actually pays for the cost Marginal tax rate = tax rate paid on the next dollar of income Marginal utility of income : the wealthier you are, the less each dollar lost to the tax collected diminishes your well-being. (Non)Recognition – once you’ve realized income there are still situations where Congress has said you do n’t need to pay taxes yet Progressive tax – average tax rate rises as income rises Proportional tax – average tax rate is constant as income rises Realization have income, but it doesn’t become taxable until an event occurs, generally when the taxpayer can claim the income in cash form Regressive – average tax rate falls as income rises Schedule approach : a sep rate schedule applies to several different categories of income and losses within each category are allowed only with respect income from that category (**we use both) 1

Upload: hoangduong

Post on 04-May-2018

212 views

Category:

Documents


0 download

TRANSCRIPT

FEDERAL INCOME TAX OUTLINEFALL ‘13, BUCHANAN

I. INTRODUCTIONa. Terminology

§7701(a)(1) – “Person” is defined as an individual, trust, estate or corp. Ability to pay : the attribute that might justify requiring some people to pay more tax than others. Average tax rate = total tax/total income Cash-flow consumption tax : simply an income tax with a deduction for savings and with the inclusion in the

tax base of amounts drawn down from savings and used for consumption, as well as amounts borrowed for current consumption.

Double Taxation – taxing the same basis twice Endowment or wage rate (the rate at which one can earn money): the opportunity to earn wealth whether or

not exercised.o Flat tax is generally not proportional and it’s appeal has more to do with simplification and ease of

administration than fairness Flow variable – defined with respect to passage of time (e.g. income as it goes up and down) Flow variable : variable whose value is determined with reference to the passage of time Global approach : income from all sources is combined and taxed under a rate schedule that applies to the

taxpayer's entire income, net any allowable losses or deductions Imputed income : the value of goods and services provided to oneself. Incidence – the ultimate burden of tax, who actually pays for the cost Marginal tax rate = tax rate paid on the next dollar of income Marginal utility of income : the wealthier you are, the less each dollar lost to the tax collected diminishes your

well-being. (Non)Recognition – once you’ve realized income there are still situations where Congress has said you do n’t

need to pay taxes yet Progressive tax – average tax rate rises as income rises Proportional tax – average tax rate is constant as income rises Realization – have income, but it doesn’t become taxable until an event occurs, generally when the taxpayer

can claim the income in cash form Regressive – average tax rate falls as income rises Schedule approach : a sep rate schedule applies to several different categories of income and losses within

each category are allowed only with respect income from that category (**we use both)o Seen in IRAs (individual retirement accounts) – amounts set aside in an IRA for retirement are

currently deductible in computing the amount subject to taxation and amounts withdrawn from the IRA account are included in the income in the year of withdrawal.

Stock variable – defined based on moment in time (e.g. bank account number) (new lawyers with high incomes but large student loans have high flow variable and really low stock variables)

Stock variable : variable whose value is not determined with reference to the passage of time Tax Base – what is being tax (can tax either the flow or the stock) Tax Gap: Revenue that we should be collecting and were not? 450 Billion or 17% owed to fed gov't was not

paid initially; 65 Billion was collected by the IRS after the fact Tax Expenditures: Revenue  a government foregoes through the provisions of tax laws that allow

(1) deductions, exclusions, or exemptions from the taxpayers' taxable expenditure, income, or investment, (2) deferral of tax liability, or (3) preferential tax rates.

Taxable unit =individual, or group, who is or are treated as a taxpaying unit in the sense that they must aggregate their income for purposes of calculating tax payable

o The broader view looks at people’s material well-being without regard to liquidityo This is so as to avoid possibly discouraging the exercise of wage-earning ability to avoid paying

taxes. To the extent: by the amount Zero bracket – amount of money, not subject to tax (is excluded from gross income and therefore not part of

taxable income due to personal deductions and standard deduction)b. Economic Consequences

i. Economists typically describe the effects of taxes by reference to income effects and substitution effects

1

1. Income Effects are changes in behavior induced by the fact that the tax reduces the money available to the taxpayer

2. Substitution effects are changes in behavior that arise from a change in the relative attractiveness of different commodities or activities

ii. Income tax should have both effectsc. Theory & Policy

i. About half of fed revenue (55%) comes from income tax1. 45% from individual income and 10% from corporate income

ii. Taxation and Ability to Pay 1. Tax scholars have long used the term ability to pay to describe the attribute that  might justify

requiring some  people to pay more tax than others2. The view that the rich should pay more than the average rate of tax, and the poor less or none, rests

on a nation of declining marginal utility of incomeiii. Taxing & the Budget

1. The general approach is to identify various exclusions, deductions, deferrals, and credits that are seen as departures from a neutral concept of income taxation; then to figure out the cost of these provisions; and then attribute these costs to various budget functions 

2. Some provisions of income tax law are designed to measure ability to pay, others are instead intended to subsidize or penalize particular activities

a. Mostly non-ability to pay provisions3. Tax payments are not a zero sum game – either someone else has to pay to cover it or some program

will not be funded or you’re going to have to raise the deficit, and if you’re okay with raising the deficit or cutting a program why not do it anyways and give a broad tax cut to everyone

4. Tax Expenditures: see chart on 17 (incomplete table) a. Three big areas: homeownership subsidies, pension contributions and earnings (employer

plans), healthcare [middle class people who vote!]iv. Tax Incidence

1. Incidence=ultimate economic burden of a tax a. Person bearing the burden may not be the same as who remitted the tax to the govt, i.e. passing

tax to customers, SHs, suppliers etcb. Burden of income tax usually falls on individual (different for corps)

2. To the extent that the tax system favors one type of investment over another, money and other resources will be shifted from the disfavored type of investment to the favored type

3. People will structure their behavior based on tax scheme and will act to lower their tax bill, people who would otherwise turn down a handout feel as though they are entitled to “their” money

a. Implicit taxes – tax code can change the take home amount even when you don’t pay money into treasury (e.g. municipal bonds can be competitive at a lower interest rate because people don’t have to pay taxes on income from municipal bonds)

v. ** Though it may be indirect, a tax break is never a freebie, taxes should be understood as compared to something (to fund public schools you either need to raise taxes or get rid of schools)

vi. Inflation 1. Inflation makes it difficult to levy the right amount of tax on investment income2. Congress required an increase in the standard deduction to reflect increases in the Consumer Price

Index so as not to penalize taxpayers for inflationd. Taxable Unit and the Marriage Penalty

i. Married people are permitted to file a joint return, so it doesn't matter who earned what; they get the lowest rate schedule

1. Married people are better off than they one would be under a system with one schedule for all because they have the best rate=marriage bonus

2. On the other hand, two earner married couples are typically worse off than if they had remained single because of the requirement that they file jointly=marriage penalty

3. Secondary Worker: the person whose income is lower and whose attachment to the workforce is weaker AKA marginal income 

a. Subject to marginal tax rates determined by primary worker

2

ii. Heads of households are unmarried person with dependent living with him or heriii. Unmarried individuals AKA single people have third most favorableiv. Married people filing sep returns have least favorable-almost always leads to increased taxes as opposed to

filing jointlye. IRS

i. IRS is not a government agency, it’s the treasury – and is politically unpopular and has no friends in Washington

ii. §7491 – Changed burden of proof to IRS once taxpayer provides any credible evidence (IRS never cared about legal burden, only about burden of production so requiring credible evidence keeps burden on taxpayer while providing for a politically friendly cosmetic change)

iii. RRA-98 – Investigated abuse by IRS found 4 cases total, also created office of taxpayer advocate, but so few complaints they’ve helped make it more efficient in other ways

iv. Acquiescence – when the court of appeals decides a tax case, the service will announce whether it acquiesces or not which acts as advice to the rest of the service whether they should rely on the outcome or whether the service will re-litigate the same issue in a different circuit in hopes of a circuit split and cert by SCOTUS

f. Compliance and Admin i. Evasion…you are on the illegal side of aggressiveness

ii. Avoidance…you are on the legal side of aggressiveness1. **Cheating on your taxes is not a victimless crime!! Affects everyone

iii. A person who fails to file an income tax return may be subject to both civil and crim penalities1. Criminal portion is very small -- difficult to meet the heightened mens reas requirement of Cheek so

prosecutors don't want to bring these cases2. Civil penalties

a. Trying to put the tax payer in the same position they would have been if they had paid the tax i. So you'll pay interest, but that’s not a penalty! You just had free use of money for the

time it took for you to repay (as if it was a loan)b. Possible different degrees of penalties

iv. Risk of being caught is high, since employers are required to file information returns1. Opposite incentives for reporting-the less you report, the less taxes you owe BUT for the employer,

the more salary they report, the more they can deduct and reduce their taxable incomev. IRS audits-some random, some because there is a greater than average probability of error

vi. Judicial Review 1. Original jurisdiction through 3 different courts AKA where you can file your case

a. your district court-->your circuit court-->SCb. tax court: special, article 1 judges on appointed 14 year terms (may have to apply different law

depending on state of P)-->your circuit court-->supreme court c. federal court of claims-->fed. circuit-->supreme court

vii. Opinion Practice and the Market for Tax Advice 1. Tax opinions are often expressed in terms of varying levels of confidence regarding the likely

outcome of litigation if a return position is challenged a. Reasonable basis: 15-30b. Substantial authority: 30-50c. More likely than not: 50+d. Should prevail: 70+e. Will prevail: 90+f. Also an obligation to report, or at least not lie, if there is NO support

g. Sources of Tax Law i. IRS Code: Title 26 of USC

1. No matter how long, it will never be complete aka loophole free2. It is written logical backwards 

a. Section 1 says apply these rates to your taxable income, but what is your taxable income (section 63)

3

ii. Treasury Regulations1. Not law, but very, very persuasive authority 2. Because there are gaps in the tax code, Treasury given the power to make interpretative rules

a. A reg. is highly persuasive guidance, but is not lawiii. Revenue Ruling

1. Similar to a judicial opinion with hypothetical facts2. Essentially advisory opinions3. Helpful and persuasive, but not as much as the Regs.

iv. Private Letter Ruling 1. Taxpayer finds a gap in the law, wants to know how the gap applies to them, and sends a letter to the

IRS to answer it2. They could create a reg for this issue, but instead they send a letter that says we comitt ourselves to

nothing, but here's the interpretation, if you follow it we won't come after you, but not necessarily if someone else does it 

a. But in reality IRS has been coerced into not going after anyone who follows any PLR even if it wasn't issued to them

v. Common law: Substance dominates form 1. You don't want hypertechnical arguments to subvert the purpose of the tax law

h. Rate Structure i. Average vs. Marginal Tax Rates

1. Overall the US fed tax system is progressive, though the degree of progression varies depending on the tax

a. SS and Medicare are regressive2. Within the individual income tax, progression is accomplished primarily by with the use of

increasing marginal tax rates-with increases in rates that apply only to higher increments of incomea. Progression is also accomplished by exempting a certain amount of income from taxation, as

well as limiting the availability of certain deductions and credits to persons with income below some threshold

3. Income – (exclusions + above the line deductions) = AGI4. AGI – (personal exemptions + (standard deduction OR itemized deduction) = TI5. (TI – credits) x (marginal r]ate) = tax due6. Alternate minimum tax 26% for first $175K and 28% for amounts over that and is payable only if

greater than the amount otherwise paidii. There is a zero rate bracket, the amount of money which people pay no taxes on

iii. §63 (b) [Individuals who do not itemize their deductions] In the case of an individual who does not elect to itemize his deductions for the taxable year, for purposes of this subtitle, the term “taxable income” means adjusted gross income, minus (1) the standard deduction, and (2) the deduction for persona exemptions provided in §151

1. Average tax rate= total tax/total income2. Marginal tax rate=% of dollar of income paid on the next dollar earned3. Tax brackets: apply different marginal tax rates to blocks of income4. Zero bracket: the income substracted from GI, to which no tax rate is applied

a. Let's say youre a fam of four i. 12,200 + 4x3900-27,800=zero bracket

5. Progressive: tax rate increases with income6. Proportional: tax rate is constant7. Regressive: tax rate decreases with income8. Deductions: standard deduction $12.2 K9. Exemptions: personal exemptions $3,900/person

iv. Federal income tax rate is the only one in the US designed to be, and actually is, progressive 1. Both in terms of marginal and average tax rates

v. Examples:

4

a. 0 % 0-15kb. 20% 15k+c. TI= 14k, tax =.1* 14k=1,00, net income=12,600d. TI= 16k, tax=.2*16K= 3,200, net income=12,800 WRONG

i. 10% on first 15000ii. 20% on (16-15k)=1k

iii. So tax is 1,500 + 200=1700iv. Net income= 14,300 

e. Cliff effect: you make more pre tax but less after taxes2. Married filing jointly with 2 kids

a. ZB= 27,800b. GI=22,000

i. TI=22,000-27,800=0 (can't be negative)ii. Tax=0, Average Tax Rate=0% (0/22000), Marginal Tax Rate=0% (22,001 they would

still be in zero bracket)c. GI= 35,000

i. TI=35000-27800=7200ii. Tax=.1*7200=720, Average Tax Rate=720/35000=2.06%, Marginal Tax Rate=10%

(what bracket theyre in-they are still within the bracket)d. GI=75,000

i. TI=75000-27800=47200ii. Tax=1785+.15(47200-17850)=6187.50, Average Tax Rate=6187.50/75000=8.25%,

Marginal Tax Rate=15%e. GI=130,000

i. TI=130000-27800=102,200ii. Tax=9982.50+.25(102,200-72500)=17,407.50, Average Tax

Rate=17407.50/130000=13.39%, Marginal Tax Rate=25%f. GI=450,000

i. TI=GI-ZB=450-27.8=422,200=107,768.5+.35(422,200-398,350)=116,116ii. Average=116,116/450,000=25.8

iii. Marginal=35%g. GI=750,000

i. Tax=233,637.20ii. Average=31.2%

iii. Marginal=39.6%3. ***Your average rate is never higher than your marginal rate

a. If you earned 72500, your marginal rate you be 25% because your next dollar would knock you into that bracket

b. High income families benefit from lower brackets i. Accounting & Deferral

i. Cash method – amounts are treated as income when received in cash and are deductible when paidii. Accrual method – items are treated as income when earned regardless when received

1. Most individuals use cash method, businesses use accrual method-more accurately reflects economic realities

iii. Money has a time value associated with it – money is more valuable the longer you can hold on to it (see table pg 39) (although liquidity or procrastination can also be factors)

1. So long as interest rates are +, TPs generally find it advantageous to find ways to defer their tax liability to future years

2. The advantage increases with interest rates, but may be surprisingly high even when they are relatively low

j. Complexity, Congress, and the Code i. Why is the tax code complicated?

1. Life is complicated2. Loophole plugging3. Political favors-it would be nice to do less of this; but easy to hide, anonymous favor

5

4. Tax expenditures....sometimes just easier to lump all these things into your taxes, instead of having multiple agencies involved 

ii. Why can't we just have everyone pay the same amount of tax? They tried a poll tax or per capita tax, in England 30 years ago and everyone flipped

1. But this doesn't seem fair, we want the tax code to reflect certain social judgments2. We want people to approve of the system when they think about it

iii. that you cannot take a tax deduction when you donate to, but the organization can operate nonprofit to engage in social welfare 

II. Income – what exactly is it a. Basics

i. Gross income: all income from whatever source derived Sec 61a 1. Except as otherwise provided in this subtitle, gross income means all income from whatever source

derived, including (but not limited to) the following items – (1) compensation for services including fees, commissions, fringe benefits, and similar items (2) gross income derived from business, (3) gains from dealings in property, (4) interest, (5) rents, (6) royalties, (7) dividends, (8) alimony and separate maintenance payments, (9) annuities… (up to 15)

2. 1.61-2: need to use available evidence3. What is income ought to reflect what we actually want to tax because we think income is a

proxy for ability to pay-who should be treated the same and who should be treated differently4. Eisner “Income may be defined as the gain derived from labor, from capital, or from both

combined”5. Glenshaw “Undeniable accession to wealth, clearly realized, and over which the taxpayers have

complete dominion [is income]. The mere fact that the payments were extracted from the wrongdoers as punishment for unlawful conduct cannot detract from their character as taxable income to the recipients.”

a. The touchstone of income questions thus became enrichment; gains are taxable (if realized and not non-recognized) whether traceable to labor, capital, or good fortune

ii. Haig-Simon Income is accrual (property goes up in value) regardless of whether it has been realized (and you’ve cashed out)

1. Y=C + S where Y is income, C is consumption, and S is savinga. You can either spend or save your income

2. Y= C+ dW where dW=change in networtha. Would require taxation of such items as unrealized asset appreciation, value of gov't services

such as education at state universities, even the value of leisure b. Wouldn't reward or penalize activities thought by Congress to merit special treatment 

3. Real world problems with HS…they don’t create exceptions, but “holes in the cheese” a. Realization v. Accrual

i. Accrual…you have gotten wealthierii. Realization…you have turned that wealth into cash

1. If we focus on the realization of income, we are not really taxing the person’s change in net worth, but rather how much they cashed out

b. Imputed incomei. Things that have fair market values that you have received or consumed, but are not

taxable1. Ex: increase in home value

c. Below-market salesi. Paying a different price for the same product (e.g. in-state tuition)

ii. This skews the FMVc variabled. Leisure

i. Leisure is often described as a consumption choice—could be making $ but instead don’tii. Its never in actually included though

e. Valuationi. Often difficult to determine fair market value

ii. How do you determine the FMV?—it’s what you would’ve paid if the tx were arms-length

6

1. 1.61-2(d)(1): if the services are rendered at a stipulated price, such price will be presumed to be the fair market value of the compensation received in the absence of evidence to the contrary

iii. Tax Equity1. Vertical ensuring that people pay only up to an amount that it would be fair to make someone in

that situation pay (what everyone situated is paying is fair for someone in that situation)2. Horizontal treating people in similar situations the same way

iv. Non-Cash Benefits 1. The definition of gross income makes no distinction whatsoever between cash and noncash

transactions a. If you didn't get taxed on these things, people would be incentivized to take jobs with non cash

benefits and wages would fall and efficiency may be reduced 2. Grossing Up

a. Old Colony : holding that an employer's payment of federal income taxation on behalf of its employee constituted income to the employee 

i. Due to withholding, it is now routine for the employer to make federal income tax payments on the employee's behalf, and yet for those payments to be included in gross income

ii. Grossing up: negotiating with your employer to pay income taxes – you have a target after tax net income

1. N = G(1-t) where N = net pay; G = gross pay; t = tax rate2. G = N/(1-t)3. If you cross an upper limit of a bracket with the gross income, it becomes more

complex4. Example:5. 80k pay, t=.2

a. GI=TI=80,000 so tax=16,0006. But 16+80=96, need to pay .2

a. Tax=3,200 more income....infinite series7. JUST reverse the income

a. G-t*G=Nb. G(1-t)=Nc. G=N/1-td. G=80000/1-.2 then G=100k

3. Meals and Lodging provided to Employees a. Bengalia

i. Petitioners are husband and wife living in HI, husband has been employed as a hotel manager, and constantly on duty, so him and his wife occupied a suite of rooms at one hotel and received their meals from the hotel 

ii. Uses a convenience of the employee test – if the lodging and meals are for the convenience of the employer then it is not taxable as income to the employee

iii. PROF: rule and holding WRONG1. Prof—majority correctly limited the opportunity for collusion between the employer

and the employee under the COE test by shifting the burden of proof away from the Service and onto them

b. SECTION 119, enacted in 1954, ended the common law realm of convenience of the employer cases by adopting a specific version of the general approach

i. Rule Excludes from income the value of meals and lodgings furnished to an employee by his employer, provided that the meals and lodgings are furnished on the employer’s business premises and for the employer’s “convenience.

ii. Meals and lodging furnished to employee, his spouse, and his dependents, pursuant to employment

1. (a)(1): Meals shall be excluded from income only if furnished on business premises (not the fanciest restaurant in town)

7

2. (b)(4): Meals must be furnished to at least half of the employees for the meals to meet the convenience of the employer standard

a. egalitarian provisionb. geared at preventing special treatment

3. (a)(2): Lodging shall be excluded from income only if it is a condition of employment that you must live on the premises

4. (b)(1): A trier of fact may consider evidence of a contract saying that lodging is for the convenience of the employee; however, such contract will not be determinative

a. this is to prevent collusion between the employer and employee** Doesn't define convenience, relies on courts and regs for that

iii. Lodging furnished to certain EDUCATION INSTITUTION employees 1. (d)(1): in the case of an employee of an educational institution, gross income shall

not include the value of qualified campus lodging2. (d)(2): Exception – cases of inadequate rent (cases when the employer charges

below “reality rent” (regardless of whether it is charging FMV to others) to its employees

a. Paragraph (1) shall not apply to the extent of the excess of i. (A): the lesser of:

ii. (i) 5% x appraised value of the qualified campus lodging, oriii. (ii) the average yearly rent paid by those not favored by the

university (non-employees)iv. non-favored yearly rent (per/month x 12)v. (B): Lesser of (i) and (ii) – employee’s yearly rent (per/month

x 12) vi. If this number is positive, this is TI

b. 119 D (2) i. You need several key pieces of information

ii. Appraised value of lodging $ 300,000iii. Rent paid by nonemployees $12,000/yriv. Rent paid by employees $3,000/yrv. 119 d 2 A i: .05 *300,000=15,000; ii: 12,000 [the lesser]

vi. So we want the excess over 12,000-3,000 [the rent paid]=9,000vii. So 9,000 are not covered by D1 so you have to include it in

your taxable income3. Qualified campus lodging

a. 119(d) applies when it is a university, and the university is providing rental apartments. However, if you are living somewhere for the convenience of the university (the employer) – such as a RA – then 119(a) applies instead.

4. Policy behind 119(d)a. The policy behind 119(d) is that when a university wants to provide its

employees with discounted housing, then the university has to provide that benefit to all people. If it does not provide discounts to everyone, then the employee will have to pay taxes on some level of income.

i. Horizontal equityii. Also, remember the employee is not living at the convenience of the

employer herec. Example:

i. Facts1. Reality rent = 7,200

ii. Analysis (119)1. (d)(2):

a. reality rent = 7,200 per yearb. rent charged = 200 per month (2,400 per year)c. below reality rent

8

2. (d)(2)(A): lesser of:a. (i): 5% x appraised value of qualified campus lodging

i. 200k is the FMV if the unit went condoii. .05 x 200k = 10k

b. (ii): for comparable units, what the university is charging to not favored people (non-employees)

i. Monthly rent = 800ii. Annual rent = 9,600 (800 x 12)

3. (d)(2)(B): employee’s renta. monthly rent = 200b. annual rent = 2,400

4. 9,600 (lesser of (i) and (ii)) – 2,400 (employee’s annual rent) = 7,200a. $7,200 would be included in TI

d. Application of §119i. Convenience of the employer: business reasons other than compensatory intent for

having the employee accept free or below-cost meals and lodging 1. Difficulty lies in determining how strong the nontax motives must be and how they

are to be demonstrated ii. Kowalski (SCOTUS) meal vouchers to highway patrols were not excludable b/c not

furnished by employeriii. Sibla (9th Cir) $3/day to participate in obligatory organized mess was excluded from

income (9th Cir essentially overrules SCOTUS but is left unchallenged)iv. RR 71-411 Employer’s convenience is most often established by proof that the

employee is ‘on call’ outside of business hours1. 119 b 1: Provisions of an employment K shall not be taken as determinative of

whether the meals or lodging are intended as compensation v. RR 75-540 Official residences of the governors of the states, and presumably the

White House also qualify as location of employmente. Business premises of the employer: circuits have split on what this constitutes for state police,

could be every highway etc i. Some courts say across the street ok, others say 2 blocks away is not 

f. Employee: rules out self employed people, but what about a SH who owns all the shares of a corp, appearing self-employed 

i. J Grant Farms Inc 1. Grant owned and operated a farm2. In 1976 he formed a corp, all of whose shares were owned by himself and his wife 3. Grant transferred or leased to the corp all of the assets used in the farming business,

hired Grant as its employee as manager, and required that Grant live in the house as a condition of employment 

4. Tax Court allowed the corp deductions for depreciation and utility costs or the house which had not previously been allowable to anyone, and Grant was not required to include any amount in his gross income by virtue of its use 

Other fringe benefits statutes American approach to fringe benefits

o Fringes started to be provided, and the if IRS doesn’t notice or doesn’t care, then the fringes get more popularo Among the most important fringe benefits are life insurance, medical insurance and payments, discounts on

merchandise, parking, company cars, airline travel, club memberships, and tuition remissions  This is a huge morale issue

Loss aversiono People are more bummed out when they thought they had something then lost it,

rather than knowing they never had it at all Raise problems of valuation, enforcement, and political acceptance

Valuation problem o Value of a benefit depend on the circumstances of that person 

9

o Through the years, some fringe benefits were excluded as a result of the Service's inaction or acquiescence

o Others were excluded by express statutory provisions, i.e. life and medical insurance o In 1984, Congress enacted 132, providing comprehensive coverage of fringe benefits 

132: fringe benefits Drew a line at the fringe benefits people have gotten used to receiving as non-taxable

o There is no commonality amongst the fringes treated as non-taxable 61 a 1 was amended to include a specific mention of fringe benefits, the main excludable ones are:

o No additional cost services such as free seating for airline employees that would not otherwise have sold outo Qualified employee discounts, i.e. department store employee discountso Working condition fringes, i.e. business use of a company careo De minimis fringes, or those of sufficiently low value to make accounting for them unreasonable or

administratively impractical, i.e. certain eating facilitieso Qualified transportation fringeso Qualified moving expense reimbursemento Qualified retirement planning serviceso Certain on-premises gym and other athletic facilities 

No additional cost services and qualifies employee discounts are subject to certain restrictions o One must work in a  line of business of the employer in which the item at issue is ordinarily offered for sale to

customers o Neither applies to highly compensated employees if the employee discriminates in favor of such employees in

determining to whom a given fringe benefit is available o fringe benefits apply to families – i.e., spouses and dependents, and now same sex couples post-DOMA

Also applies to retired and disabled employees and surviving spouseso Can also make a reciprocal agreement with another business

(b) no-additional-cost serviceo e.g., empty seats on airplanes

letting an employee fill an empty seat is of no additional cost service to the employer the employee receives something of value, but 132 allows the employee to exclude that

from the value of their incomeo (1) “in the ordinary course of the line of business”

intended for corporations with multifaceted businesses if you are a programmer for a corporation that happened to have an airline, you

CANNOT get the free airfare dealo (j) “boys in the boardroom” provision applies

(c) qualified employee discounto as long as the employer sells something to an employee at cost

in the case of services, the Code simply institutes 20% employee does not have to include the difference between cost and retail, even though

they are receiving a benefit o (4) “in the ordinary course of the line of business”

Same as 132(b)(1)o (j) “boys in the boardroom” provision applies

(j) special ruleso (1) exclusions under no-additional-cost service and qualified employee discount fringes apply to highly

compensated employees only if there is no discrimination goes against the boys in the boardroom problem for our purposes, highly compensated means highly compensated

(d) working condition fringeo It can be excluded when it is provided by the employer as fringe benefit if an employee could have

deducted it as a business expense itself (e) de minimis fringe

10

o most important fringe of all does not mean it is a small fringe, it means that it is an annoying fringe

accounting for them would be unreasonable or impracticable o not about “lines of business,” about annoyance

(f) qualified transportation fringeo (1) you can exclude from your income, the costs of a

commuter highway vehicle essentially employee vans taking employees from satellite parking lots to their offices

o More than 6 seats (excluding the driver) and 80% if mileage for transporting employees to and from work where ½ of the seats are filled

a transit pass qualified parking, bicycle commuting -- from your income

o (2) limitations on exclusion – an exclusion shall not exceed $230 per month for commuter highway vehicles and transit passes $230 per month in the case of qualified parking the applicable annual limitation in the case of any qualified bicycle commuting reimbursement

o this fringe is here to allow people to exclude commuting costs from their taxable incomeo Hypo on page 59

The more the transportation looks like a rolling office, ie phone and lamp, the more likely this qualifies as a non taxable fringe benefit-he's working the whole time!

1.61-21(b)…taxation of fringe benefitso Valuation

The treasury has published detailed regulations on fringe benefits regs 1.61-21 and 1.132-1 to -8 FMV if you can determine it If you can’t determine FMV, then there may be a safe harbor rule to turn to

Sometimes it’s difficult to determine FMV. So if you follow a safe harbor rule, you can avoid an IRS challenge.

o (b)(5) chauffeur services: use what the limo company paid the chauffeur Frequent flyer credits

o When credits are earned from personal travel, they are best through of as reductions in the price of the flights and there should be no tax consequence

o But its different for business trips, you can use these benefits for your personal trips-experts agree this is income  

o For many years the IRS made no effort to impose tax liability in these circs  Finally threw in the towel in 2002 stating that the IRS will not assert than any TP has understated his

federal tax liability by reason of the receipt or personal use of frequent flyer miles, etc IRS says if you get them for opening an account, you have to pay taxes; if you get them for

using your card or for taking a promotional trip [these are more like rebate], then you don't have to pay for taxes

Health Insurance o Employers are allowed to deduct the cost of medical insurance that they buy for their employeeso The benefits received by employees are excluded from their from income (106a), and extends to spouse and

dependents o Also excluded from wages for the purposes of determining payroll tax o Self employed TPs are allowed to deduct the costs of medical care under 162 (1)o The general effect of the exclusion of health insurance from income is that the government subsidizes a

portion of insurance costs, leading to employees trading up on health insurance policies  What if 132 was around when the Benaglia case happened?

o Food wouldn't count, he'd have to pay taxes (because its an additional cost service...except for the food needed for him to stay late)

o But if the rooms he stayed in were going to be vacant any way, then he could have argued that to be tax freeo BUT wait this is moot because of 132 J

11

These things are not being offered to other employees, highly compensated employee ruleBenaglias lose

Cafeteria plans

125: cafeteria plans Another kind of fringe benefit

o A plan under which an employee may choose among a variety of noncash nontaxable benefits or may choose to take cash 

Provide nontaxable fringe benefits to those employees who want them without disfavoring employees who have no need for them 

o Increases potential use of non taxable fringe benefits by removing the element of employee envyo Limits the fringe benefits that can be included in a cafeteria plan and imposes a nondiscrimination rule

Include group term life insurance, dependent care assistance, adoption assistance, excludable accident and health benefits, elective contributions for 401k

(a): the fact that you have a choice among benefits does not turn them into cash and make them taxableo 132 (a bunch of isolated benefits that you may receive) v. 125 (an employer has to specify a cafeteria plan)

(d): you can choose fringe benefits from the cafeteria plan tax-free, but if you opt out, take the cash, pay taxes on that income

(f): inclusion by reference (anti-abuse provision)o other Sections of the Code tell you what can be included in a cafeteria plan

Key examples of what CAN BE INCLUDED in a cafeteria plan Group-term life insurance Dependent care assistance Adoption assistance Excludable accident and health benefits Elective contributions under a qualified cash or deferred arrangements under 401(k)

The idea of a cafeteria plan is to allow employees to pick and choose all, some, or none of the benefits provided by the employer

If you take all of the benefits, you will have a lower taxable incomeo So, there is no horizontal equityo But, there are psychological aspects to this

You can personalize the fringes to your own situation Employee envy

The fact that you can opt-out should allegedly reduce the element of employee envy Use-it-or-lose-it rule

o If you don’t use the whole benefit up, you just lose it (so employee must plan) This stems from an incorrect estimation of what you may need in a flexible spending account

People go in for all kinds of unnecessary medical procedures on March 30 or 31o To cure these problems, Prof suggests that Congress simply set limits rather than attempting to prevent people

from overspending on the preferred items by raising the risk associated with overestimatingo If you don’t use it, the funds go back to the employer

The employer may pocket the excess funds Or, the employer may redistribute the excess funds across all the employees

Cafeteria Plans Example: o C: 50k salary

Takes no 125 fringes  Tax=10k, Net=40k-5,000 (to make meaningful comparison)=35,000

o D: 50k salary 5k in 125 fringes tax=.2*(50-45k)=9,000, Net=36,000

o t=.2o Makes cost to employer the same whether the employee takes fringe or  not

12

Another Approach to ValuationTurner (TCM 1954)

Question Presented : the amount which should be included in income because of the winning of steamship tickets by answering a question on a radio show

o Both sides agree that its HS income, that its section 61 income, etc They are willing to admit this

o The only question is what damages should be Court decides that the tickets are income of 1400 Reasoning: don't provide any reasoning....notes say that they split the difference between Turner

and IRS valuations Suggests a subjective approach to valuation…incorrect

McCoy v. Commissioner A guy won a Lincoln worth 4.4K, traded it in for a station wagon worth 2.6 K + 1k cash Court held that the amount includible in income was 3.9K

Rooney The use of any such subjective measure of value as is suggested is contrary to the usual way of valuing either

services or property (this is right)

Rule You want to look for objective manifestations of value

o Important distinction Opinions and the like can affect fair market value if people put their money where their mouth is What we don’t want to do is take into account people’s unique situations

Windfalls and perks

Treasure trove 1.61-4: treasure trove, to the extent of its value in US currency, constitutes gross income for the taxable year in

which it is reduced to undisputed possession If you gave back such a windfall immediately, as in the case of the record-breaking home-run balls, you are

treated as if you never possessed it at all. There is a realization requirement for found treasure.

o Only when you sell it and turn it into cash are you taxed on it. HS income but not taxable income.

Taxes and Space Company should gross up and pay for the taxes of the gifts they give because they get free publicity So if you don't turn it into cash but you use it then its taxable

o ie trip to space, so you’re going to need a gross up Gross up example

o Car costs 35,000o What would the taxes be??o 35000=G-tG=G(1-t)=> 35000/(1-t)=Go Lets say its 25% so 35000/.75=>about 48K so you give 13K in cash 

IMPUTED INCOME People may use their property or their own services to provide benefits directly to themselves or to members of their

households  o The benefits they derive are not part of any commercial transactions, and experts refer to these kinds of

benefits as imputed income  Benefits that are not part of a commercial transaction and are therefore generally not thought of as income for tax

purposes are generally not included in a TP’s taxable incomeo Though failure to include them results in problems of fairness and economic rationality

13

Property (other than cash) – increase in value of a house is not taxed so long as it is unrealized and if passed at death with a stepped up basis will never be taxed and so the investment will escape taxation shows a congressional preference for home ownership and that tax system will influence actions

Owner-occupied home: the imputed income is simply the rental value of that home  Some assert that the treatment of homeowners discriminates against renters, but there are significant

tax benefits to landlords, mainly in the form of deductions for cost recovery that may be unrelated to economic reality 

Presumably these benefits are passed onto tenants in the form of reduced rent  The real reason people are not taxed on imputed income from home ownership is because it would be

impractical to do so because of problems of valuation  Services – the benefit of the services that one performs for oneself is not taxed

o Service of homemakers are of substantial value and are not evenly distributed among households  One way to make this more fair is the section 21 and 129, allowing credit for child and household

care and tax free employer reimbursement for childcare expenses, combined with 125, allowing employers to offer employees a choice between tax free benefits and cash 

o The value of services that one performs for oneself in creating human capital  I.e. studying to become a doctor

Physic income and leisure – there is a tax on the earnings, but not the benefit of leisure that one “buys” by not working – so there is a tax benefit to being underemployed, you can buy leisure time with pre-tax dollars

BarteringRev. Ruling 79-24 Barter income IS taxable Valuation

o Section 1.61-2 d 1 provides that if services are paid for other than in money, the FMV of the property or services taken in payment must be included in income

If the services are rendered at a stipulated price, such price will be presumed to be the FMV of the compensation received in the absence of evidence to the contrary 

o FMV If the FMVs are different, you have to look at the FMV of what the person received in order to

decipher their taxable liability As an administrative choice, you almost always anchor the work of the thing easily valued to

the thing not so easily valuedo With a presumption for stipulated value

This value can be challenged by the IRS with affirmative evidence of their own Other examples:

o Ownership timeshare condos exchanged for a week  IRS has not ruled on this

o Exchanges of babysitting income never gonna get taxed in the real world even if its HSo Pool resources to create commune 

It’s a family so it’s an exchange of services with family so the tax code doesn’t touch Hypos

o Situation 1 Barter club Lawyer provides a house painter trade with legal work in exchange for the painter to paint his house

The FMV of the services received by the lawyer and the painter are includible in their gross incomes

Tax analysis Lawyer values his work at $1,000

o Lawyer is taxed on $500 (received) Painter values his work at $500

o Painter is taxed on $1000 (received) Situation isn’t normal as this is an example of a crappy deal …if the FMVs are different, you have to look at the FMV of what the person

RECEIVED in order to decipher their taxable liability14

o Situation 2 A professional artist provides an owner of an apartment building with a painting in exchange for 6

months rent The FMV of the work of art and the six months of FMV of the apt are includible in

gross incomes Tax analysis

6 months worth of rent…FMV easy to find piece of unique art…FMV not easily found As an administrative choice, you almost always anchor the work of the thing easily

valued to the thing not so easily valuedo This becomes even more difficult when you have two things that are not easily

valued. Then, you just hire and appraiser.

6045: Barter Clubs Requires information reporting by any “barter exchange” 1.6045-1(a)(4): the term barter exchange does not include arrangements that provide solely for the informal

exchange of similar services on a noncommercial basiso The IRS does not want to get into trivial matters, even if they are HS income.o This sometimes applies to trivial matter with commercial bases as well.

Credits from a barter club...pile on a bunch on them but don't have to include them in gross income until you cash them out

WINDFALLS AND GIFTS

Punitive damages

Glenshaw Glass (US SC 1955)

Question Presented: Whether money received as exemplary damages for fraud or as the punitive 2/3 portion of a treble-damage AT recovery must be reported as gross income 

The Court holds that punitive damages ARE includable in taxable incomeo Nothing limits the definition of gross income under §61

Punitive damages falls under “any source whatever” SC sets super precedent (super precedent…decisions that is entrenched and if changed

would be a big deal)o Income has never been limited before

§104(a)(2): personal injury recoveries Compensatory and punitive damages are excluded from taxable income when it is a personal injury case

o Even though they are HS incomeo Theory…all you are doing is making someone whole

But, this doesn’t really draw a distinction to the GG case Mental or psychological injuries are not covered, just physical injuries Pain and suffering is not taxable

o So when, through a suit, you get something non-punitive that is non-taxable, like pain and suffering YOU don't want to pay tax on it

o If you got lost wages-then that’s taxable, if you got punitive then its taxable ***Need to look for these explicit exclusion sections to not have to tax

Gifts Gift: the Basic Concept

o Gifts have always been excluded from taxable income under what is presently section 102 o Exclusion extends to gifts (such as cash) that involve no valuation difficulty, and is not simply a de minimis

rule of administrative convenience; it applies to large gifts as well as small 

15

o Things to know: If you receive a gift and it’s really a gift, you don't have to worry about income tax or gift tax Gift tax is paid by the giver not the recipient 

Under the gift tax there is an exclusion for gifts up to 14k o Example on bottom of page 82

D gives E 30k; her income is now 70k and his is 30k for income tax reasons As if he was an employee

D gives E 30 k; her income stays 100k and his is considered 30k As if he was hired to do personal services

No income to E and no deduction to D  This is the real rule on gifts

§102: gifts and inheritances (a): Gifts are excluded from taxable income, but does not income from property received or where the gift is of

income from property (b): you can exclude the initial gift, but not any income you derive from that gift

o e.g., if you get stock dividends as a gift Example (p. 16 (Notes)) Three possible ways to deal with gifts:

o Donna 100k Edward 30k (a) Income to E, deduction for D: [70k GI Donna; 30k GI Edward]

E provides non-personal services to Do How business expenses are treated

(b) Income to E, NO deduction for D: [100k GI Donna; 30k GI Edward] E provides personal services to D

o E.g., E cooks for D How personal expenses are treated

(c) No income to E, NO deduction for D: [100k GI Donna; 0 GI Edward] THIS IS HOW WE TREAT GIFTS…allowed by fed income tax Gift as long as transfer is a “gift”

o Precisely because E did not work for D E works, in some degree, in each of the other two situations

Defining “gift” (this is the hard part)o Duberstein (US SC 1960)

Berman called D one day and said the info he had given him was so helpful he wanted to give him a present and gave D a car 

B later deducted the Cadillac as a business expense on its corporate income tax return IRS thought it was renumeration for services rendered, not a gift

o No 506, Stanton Stanton worked for a church, making 22.5k a year When he resigned the church gave him a gratuity of 20k because the church really liked him and

wanted to give him a gift o Holding:

The D Cadillac was a recompense for D's services, or an inducement for him to be of further service in the future 

Court is in disagreement about Stanton; district court later found that the money in Stanton was a gift, affirmed on appeal

o RULE: Gift, statutorily: proceeds from a ‘detached & disinterested generosity, out of affection, respect, admiration, charity or like impulses.”

Court should look at the intent of the transferror not on the transferee Court basically saying we don't know what a gift is! If it looks gift it’s a gift

o Factors (1) “Detached generosity”

Detached and generosity seems to be an internally inconsistent phrase (2) “Affection, respect, admiration, charity or like impulses”

16

At least the court is getting somewhere here The transferor has to feel “happy”

(3) Transferor’s “intention” The stance of the recipient is irrelevant to determining if it’s a “gift” for tax purposes “Gift” determination does not rest on the donor’s conduct, but rather his intention

o Objective/subjective test We are reading their mind to get an objective determination of the donor’s subjective intent Court will not look to language expressing that a “gift” is a gift as outcome determinative Court will look at the totality of the circumstances Whether a gift is from employer to employee is relevant, but will not determine the court’s

outcome [LATER CHANGED BY 102c)] Whether a gift is given by a corporation is relevant, but will not determine the court’s outcome Whether a gift is deducted is relevant, but will not determine the court’s outcome

o Gifts defined as “gifts” are excludable in the recipient’s tax income [also CHANGED by 274b]o Standard for gift cases in the future

If there is an appeal, anything short of “clearly erroneous” will lead to an affirmation “Clearly erroneous” is highly deferential to triers of fact…wrong because legal finding

seems to be getting the standard of review afforded to factual findingso Pertains to facto Trial courts can handle these kinds of situations better than appellate courtso Reasonable minds could differ

Congressional action (post-Duberstein) §102: gifts

o (a): Gross income does not include the value of property, acquired by gift bequest, devise, or inheritanceo (c): value of gifts from an employer to an employee is not excludable from taxable income

this is about gifts to employees from companies In these situations, the employer may deduct the value of the gift and the employee must

include the value of the gift in his income.o We just treat such a gift as compensation.

274(b): businesses are not allowed to deduct expenses for gifts to individuals to the extent they exceed $25 (defines ‘gift’ by reference back to 102)

o this is about gifts to individuals from companies This standard governs the Duberstein situation

If business gift, the business can deduct no more than $25. The recipient, however, may exclude the entire value of the gift from his income.

o This disincentives business gifts. If not gift, and is instead determined to be an ordinary and necessary business expense, it is

deductible to the gift giver under 162. For the recip, however, the value of the gift is includable in their income.

The determination of a gift for one party would hold true for the other party as well.

Further defining “gift” Harris

o Facts: Kritzik was a wealthy widower partial to the company of young women, including twins Conley and Harris; K gave C and H each more than 500k over the course of several years

No one paid tax on this money, and C and H were convicted of evading their income tax o Only Crim case—SOR = Sufficiency of evidence, in light most favorable to gov’to A person is entitled to treat items from lovers as gifts as long the relationship consists of something more

than specific payments for specific sessions of sex to treat it as a gift, you could arrange for sex on a non-transactional basis

o Doesn't matter if she thought she was earning what he was giving to her, what matters is what HE thought (could have thought she really loved him)

We need to look at his intent...these letters can show us that!

17

In the case of married couples, taxability of transfers of money does not hinge on the application of the “detached and disinterested generosity” test of Duberstein

Tips and unusual gifts Tips

o 1.61-2(a)(1): a tip is included in taxable income because it is compensation for services Regulation 1.61-2:

Discussing how many restaurants esp chains now have "team tips" and share with the whole team

Employer is required to give employee tax return information which includes information on tips

Safe harbor rule: Now waiters can say tip income is 10% of the business you did...low average 

o With regards to Duberstein, the unusual nature or unique motives of a tip may cut in favor of counting it as a “gift,” rather than if the tip was just generally provided as additional compensation for the given transaction

But it also may not cut that way Tipping a minister for performing a wedding; minister must include in TI Gamblers who tip out of compulsion or superstition: dealers/servers must include in TI Scholarships: excluded from TI

o Section 74 created a rule requiring inclusion of such benefits in gross income with minor exceptions for charitable contributions

o Section 117 leaves only a limited exclusion for scholarships provided to degree candidates Only for the portion used for tuition, fees, books, and supplies and required equipment 

Surviving spouses – payments from corporations to surviving spouses [5 factors to determine if it’s a gift] (1) payments made to wife and not to estate; (2) no obligation of corporation to pay anything additional; (3) corporation derived no benefit from payments; (4) wife performed no services for corporation; (5) services of husband had been fully compensated

RR 76-144 – Government Payments:o Traditional welfare payments and other government payments are generally not excludable under 102 but

rather not within the contemplation of income under 61 Unemployment is now fully includable under 85 Under current law, the treatment of SS benefits depends on the TP's adjusted gross income,

augmented by one-half the benefits received, tax-exempt interest and other items Bonuses -- starting with most likely to be included as income (1) regular bonus from employer, (2) irregular bonus

from employer, (3) regular bonus from 3rd party, (4) irregular bonus from 3rd party

Transfer of unrealized gain by gift while the donor is ALIVE Section 61 a 3 includes in gross income gains from dealings in property Section 1001 provides that the amount of the gain is the excess of the amount realized over the adjusted basis 

o The adjusted basis is the basis, defined under 1012 as cost adjusted as provided in 1016o There is an exception in 1015 for property acquired as  gift 

Generally the donee's basis is the same as the donor's basis aka transferred basis 7701 a 43o Exception: if at the time of the gift the donor's basis is greater than the FMV of the property so the donor

would have a loss if he or she sold the property, then for purposes of computing the donee's loss on any subsequent sale, the donee's basis is the FMV at the time of the gift

Taft (US SC 1929)o Facts: A purchased 100 shares of stock for 1k, then sold them to B when the FMV was 2k, B sold them for

5k. IRS says B must pay income tax on 4k, B says only on 3k-the appreciation during ownershipo Held: Donee has to pay tax on entire realization even though some of it was “earned” before she got the

property – both because the law is clear and because the receiver of a gift is still better off Donee assumes the place of his predecessor

Carryover basiso Threshold issue: Is the property you are receiving a gift?

18

o 1015(a): basis of property (not cash) acquired by gifts you go back to the last person who paid money for the property that you are getting as a gift future gain…carryover basis (you use what the original purchaser paid for that property) future loss…FMV at the time of the transfer

This rule somewhat clumsily limits carryover loses and accompanying strategic tax moves

o What Congress said we we're going to stop family members giving each other property at a loss so that the receiver can get a better tax advantage than the lower income gifter

o Calculation example Original owner paid = $1500 FMV at transfer = $1000 (1) Recipient sells for $800; income = 800 (sale) – 1000 (FMV at transfer) = - 200 ($200 loss can be

subtracted from his income) Loss calculation (negative value when you subtract FMV)…subtract money from taxable

income (2) Recipient sells for $1600; income = 1600 (sale) – 1500 (carryover basis) = + 100 ($100 can be

added to his income) Gain calculation (positive value when you subtract carryover basis)…include money in

taxable income (3) Recipient sells for $1200

carryover…1200 (sale) -1500 (carryover basis) = -300o no gain

FMV…1200 (sale) -1000 (FMV) = +200o No loss

Neither loss nor gain calculation (negative value when you subtract carryover basis AND positive value when you subtract FMV)…thus, no tax consequence

o Policy behind carryover basis rule Gain side…progressive rule

You are giving away something of value, and also ridding yourself of a potentially large tax liability

Giving it away does lose you money, but some people make tax motivated decisions One of the things that this does encourage is downward distribution of wealth

Prevention of losses side…regressive rule A poorer person may transfer gifts up to a wealthy person in order for the wealthier

person to avoid tax liability o Not a policy generally espoused by Congress

Hypotheticalo 100k loss in real estateo May transfer this losing property to a wealthier person who would like to be able to

deduct the amount of the “gift” from his GI …1015 does not force transfers of wealth to just the poor

o incentive to move assets in the right direction is removed The hole…no gain or loss

o Remember: 1015 ONLY applies to gifts, so if there is an arm’s length transaction, the person receiving the property will take basis based on FMV rather than donor’s basis – so if shares were transferred as compensation, the basis would be FMV and payer would pay tax based on “sale or disposition” of property

Double Taxation  No rule against double taxation Double taxation means taxing the same base more than once

o Buy an item at the store at it is $100o Taxed at sales rate and if it was taxed again as a separate taxation 

Asking how many times a dollar has been taxed is a meaningless question because money gets reused  Is double taxation twice as bad?

19

o Doesn't tell you anything! You need more information 

ESTATE TAX: Transfers of unrealized gain by gift at death

Estate tax o Meant to interact with gift tax, known as the generation skipping tax o Designed to tax transfers of wealth at the death of the giver

Threshold issue: Is the property you are receiving a gift?o Under 1014, the basis of property acquired by reason of death is the FMV on the date of death or at the

election of the executor or adminstrator under 2032 on the optional valuation date o The basis is either stepped up or stepped down from the decedent's basis to the DoD value (usually see

stepped up) §1014 (a): basis of property acquired from a decedent

o (1): basis of property is stepped up (or stepped-down) to the FMV of the property at the date of the decedent’s death (2032 provides an optional valuation date of six months after death)

Way different from carryover basis Much more favorable Example

o 1m at decedent’s deatho 1m when you received ito you sell at for 1m the next day, then taxable income is 0

Estate tax comes in to compensate for this lack of taxable incomeo Estate tax

When a person dies you tax the value of their property

o take out the expenses of dyingo take out charitable gifts directed by the estateo take out $5.25m zero bracket

When the first spouse dies, the estate can be transferred to the surviving spouse tax free

When the second spouse dies, the estate can be transferred to beneficiaries with a $10.5m zero bracket

Marginal tax rate = 40%o Policy rationales for an estate tax

(1) Highly progressive Only high-wealth people pay it Satisfies vertical equity in spades

o Only those capable to pay it do so (2) If you collect money from this tax, you don’t have to collect as much from other taxes

The estate tax allows you have lower rates on income, allows you to have 1014, etc. (3) Estate tax serves as a back-up tax on capital gains

estate tax indirectly taxes capital gains that 1014 would let go untaxed forever (4) Encourages gifts to charity

charitable gifts taken out of an estate are not subject to estate taxes If you hold onto property long enough there will not be income tax liability on the gain

1014 creates an incentive to hold on to property and not sell it Is it double taxation with the gift tax which taxes transfers of wealth before tax? No

People argue that all the money is this estate has been earned and already been taxed once and now when I die I will be taxed again

o Buchanan says this isn't the right way to think about ito You're not taxing the dollars, your taxing the transmission of an estate (or large

gift)...different base than income tax The only way to ask whether the estate tax is good or bad is to ask what would we do in its absence?

20

**Buchanan is against repealing the estate tax and thinks we should tax more of the estates that are currently not taxed

LOANS AND DISCHARGE OF INDEBTEDNESS

Loans Equity v. debt 

o Very different from legal perspective o Borrowing money is like selling a bond to someone

Ruleo When you take out a loan, even though you have money coming in, because it has an offsetting liability, you

do not need to include that money in your income.o When you pay back the loan, you do not deduct the payments that you makeo Section 61 a 12: the rationale

Loan proceeds do not improve one's economic condition because they are offset by a corresponding liability (doesn't increase net worth)

Interest payments are taxable income to the lendero In this way, a loan operates as a savings accounto Principal is not income

Are interest payments deductible for borrowers?o Interest payments on loans are deductible to business but not individuals

Policy: Congress decided Americans were spending too much money and getting into too much debt We ASSUME that people actually make good on their loans

o If we don't know for sure if you're gonna pay it back so we should add 200k to your income and then deduct repayments

What if someone loans you money (a bond) and then just tears up the piece of paper o That's discharge of the indebtedness, and therefore you have income

If the borrower defaults, the loss is deductible to the lender 

Discharge of indebtedness “cancellation of debt” = “discharge of indebtedness” Discharge of indebtedness income (DOI income) becomes relevant when loans are not paid back

o We trust that you are going to pay back the loan until we decide that we cannot trust you KEY…The driving factor in determining DOI Income is whether the principal loan is paid in

full 61(a)(12)

o “income from discharge of indebtedness” if a debt is discharged, then the borrower essentially has received income equal to the amount of the

indebtedness

Kirby Lumber (US SC 1931) Facts: Kirby issued $12M in bonds in July 1923; later than year, they extinguished $1M of principle by paying

$826k. IRS claims the $138k is DOI income. D did not have to pay tax when they sold bonds because it was a loan, but when bought back bonds at lower price it

was DOI income and thus taxable income Discharge of indebtedness income really is income

o Income is included in the year of the discharge, not the year of the loan Kerbaugh-Empire Co case

o The TP had borrowed money repayable in marks or their equivalent for an enterprise that failed

21

Court found it was a profitable exchange on the currency exchange market, there was no DoI o Kerbaugh Empire muddies the waters because it seems to say in order to answer the DoI question, you need

to know what happens to the business Courts have not treated this as true. They make Kirby as simple as it seems; fate of an enterprise is

irrelevant.

108: income from discharge of indebtedness Congressional cure to KL

o Section 108 contain elaborate rules relating to discharge of the indebtedness of insolvent debtorso “One should not hit a person when he is down” theory

DOI not considered income if o Bankrupt (linked to bankruptcy principles)o Insolvent (you can be insolvent but not bankrupt if you haven't filled the bankruptcy papers...all law students

are insolvent!)                              (a)(1):

o (B): if your net worth is negative then you are insolvent, and if you are insolvent you do not have to include discharge of indebtedness income in your taxable income

o Qualified farm indebtedness (farmer doesn't need to be insolvent or bankrupt)o (C): you can be a farmer who is not in bankruptcy and who is not insolvent, and you do not have

to include discharge of indebtedness in your income, so long as the farm indebtedness qualifies Congress loves farmers

o Mortgage forgiveness Section 108 h provides relieve for TPs who would otherwise recognize income from the discharge of

indebtedness when mortgage lenders foreclose on the TPs principal residence  Statute provides for an exclusion of up to $2M of forgiven qualified principal resident indebtedness Added after financial crisis when a whole lot of people are losing their homes

You give up your underwater house, may not have been necessary because people were prob insolvent or bankrupt anyway

Probably won't be extended past 2014                     o (d)(1): for the purposes of 108, “indebtedness of taxpayer” means any indebtedness:

(A) for which the TP is liable, OR (B) subject to which the TP holds property

o (e)(5): purchase price adjustment after the fact, we act as if the price was different in the very limited situation in which the person you bought the good from also financed the loan, the

amount which you settle the loan for in not considered DOI Income Department store credit card situation:

You say this cost 1000 dollars, but then you both agree to pay it off for 900 So instead of 100 in DOI income, tax code says let’s just say that it’s as if it was on sale and

they sold it for 900 Increasingly not available as department stores vanish

o 108 (f): Student loans Loan forgiveness program relief (f)(2): excludes cancellation or repayment of student loan that is made contingent to work on

charitable or educational institution ** When you think about it, it makes sense why DOI should be considered income, but it also makes sense why there

are so many exceptions as a matter of policy

GAINS AND LOSSES FROM GAMBLING The Basic Rule: all gains are taxable but losses are deductible only to the extent of gains from the same taxable year 

o 165(d): wagering losses 1.165-10: You pay taxes on net winnings, but you cannot deduct net losses

You can only use losses to offset winnings, however you cannot make deductions for additional losses

22

The IRS has ruled on comps and the Tax Court has said the taxpayer can offset the comps as if they are winnings from gambling.

Comps are considered gambling income. Gambling income can be offset by gambling losses WITHIN THE SAME YEAR [IRS is rigid

about the year] Theory:

o Not just punishing gambling for moral reasons--isn't having fun at a casino and spending $100 and losing it the same as $100 non-deductible consumption elsewhere?

o Gambling transactions are place in a separate basket from all of the TP's other transactions to ensure that no net loss from the basket will be deductible 

Basketing rules give unfortunate prominence to the question of which transactions fit in the basket  Jasinski: purchase of high risk junk bonds is not gambling

Enforcemento Since gambling is a cash business, enforcement is a problem o For relatively large transactions, 3402q requires that the race track or other payor withhold taxes at 20%o Payors usually only have to report winnings of $600 or more to the IRS 

But just because you win less than $600 doesn't mean you don't owe taxes  Misconceived Discharge Theory: Section 108

o Suppose Z borrowed $10k to finance her law school education and her employer pays the debt on her behalf as a Xmas bonus

o Should be viewed for tax purposes as if the employer had paid Z a cash bonus of $10k and she had used her money to pay her debt

Should be taxed under 61 a 1 not 61 a 12o Indirect payment or economic benefit might best capture the essence of the transaction 

Zarin (3rd Cir. 1990) Facts: Zarin appeals from a holding that he recognized $2.9 million in income from discharge of indebtedness

resulting from his gambling activities and that he should be taxed on the incomeo Zarin was a high roller, and despite an investigation by Gambling enforcement, casinos continued to give

Zarin lines of credit and he became a compulsive gamblero Zarin was finally cut off when he was giving casinos checks with insufficient funds, checks totaled $3.4

million  Resorts sued Zarin for the 3.4 million, they settled for 500k

o IRS said that Zarin had recognized$ 2.9 million of income in 1981 from the cancellation of indebtedness that resulted from the settlement (3.4-500k) so he owed 70% of that in taxes 

o Majority holds in favor of Zarin MAJORITY GETS IT WRONG: Law is clear that D should have had to pay taxes on DOI, but that is too

inequitable for court and so they find ways around ito 108 d 1: The problem with using this definition of indebtedness

61 includes DOI is income, but 61 does not define indebtedness and so court looks to 108(d)(1) which refers to a charge on which the taxpayer is liable or subject to which the taxpayer holds property and finds D satisfies neither of these

Majority: he's not liable because the debt is unenforceable because the casino made you a loan in violation of NJ law

o Buchanan : Zarin said he was going to pay it in full after he argued that it was unenforceable...therefore it could be a K between private parties and it would be enforceable OR

Majority: chips are just an accounting method to prove indebtedness...but aren't they trying to show that there's no indebtedness??

o Court says that chips were Resorts property and Zarin couldn't use them to buy whatever he wanted...but then they said the chips were not Resorts property when he had them WHAT?!)

o Contested liability doctrine

23

If a taxpayer, in good faith, disputed the amount of a debt, a subsequent settlement of the dispute would be treated as the amount of debt cognizable for tax purposes and any excess over amount determined to have been due is disregarded for both loss and debt accounting purposes (judicially created). This particularly involves situations concerning illiquid damages.

DOI= Original loan-Payment Amount--->IRS says 3.4million-500k; Majority says 500k-500k therefore DOI=0 because it is a contested liability [would not be zero only if you don't pay back your settlement amount]

o Buchanan: This is not proper here because it would essentially read DOI income out of the Code.

Dissent: Zarin never reported the 3.4 million as income because he recognized it was a loan that he had to repay o Income not earlier recognized should be recognized when the debtor no longer recognizes an obligation to

repay and the creditor has release the debt or acknowledged its unenforceability o ** Every other circuit has repudiated it, good law is the opposite of what the majority said

Alternatives: as a matter of equity, you can figure out an equitable arrangement without messing up the law o IRS could have been ordered by the court to do a compromise aka make him less...so this means YES he has

DOI income but no you can't get  all the moneyo OR through the insolvency exception from section 108

GAIN ON HOME SALES Congress has turned to the tax code to curry favor with America's homeowners Average homeowner only lives in their home for 6 years in the US

o If you let your house appreciate enough and then their gain dwarfs their income they may be pushed into a higher tax bracket

o Also do we want to tax someone on their only appreciable asset?OLD RULE: 1034 – If income on sale of home, taxpayer can exclude that income if she buys a home of greater or equal value

within 2 yearso Well just treat it like you’ve been living in the bigger home the whole time

121 – At age 55, one one-time 125k exclusion, even if not used to buy home of greater or equal value within 2 years Policy for OLD rule

o Government was creating this new ethos of home ownershipo The path cut by Congress

Continue to buy nicer houses until you want to cash out If you continue to follow this path of owning a house – then considered the ultimate middle

class asset – then you will receive a nice exclusiono Difference in incentives comparing old rule to new:

Old rule encouraged people to move into bigger homes whether they needed it or not. Got built into the psychology that when you move you move bigger

Difficult for people moving from high income areas to low income areas Also made people all sell their houses the year they turned 55

NEW RULE:121: exclusion from a gain from sale of principal residence (b)(1): Singles

o Requirements for singles to get 250k exclusion: (1) Used home as principle residence for 2 (in the aggregate, not consecutively) of the last 5 years

o Any GAIN over 250k is taxed at 20% [capital gain tax rate] (b)(2): Couples filing jointly

o Requirements for couples filing jointly to get 500k exclusion: (1) either of them have owned the home for 2 (in the aggregate, not consecutively) of the last 5

years, AND (2) both of them have used it as a principal residence for 2 (in the aggregate, not consecutively) of

the last 5 years, AND (3) they are not subject to any other exceptions

24

o Any GAIN over 500k is taxed at 20% Reg. 1.121-1: “principal residence”

o Totality of the evidence test What you need to do is look at all of the circumstances that indicate whether it is this TP’s personal

residence Does the TP have other residences? If no, then… Where are they registered to vote? Where is the car registered? Where are the kids in school?

o You can only have 1 principal residence (c)(2)(B): “unforeseen circumstances”

o You can get a proportional exclusion if you don’t meet the above requirements, but you moved you changed your place of work, for health reasons, or unforeseen circumstances.

o What does “unforeseen circumstances” mean? “such sale or exchange is by reason of a change in place of employment, health, or, to the extent

provided in regulations, unforeseen circumstances.” Congress is trying to limit people who simply want to move prior to their two years being up

Reg. 1.121-3 Facts and circumstances test

o Safe harbors (these instances are considered unforeseen circumstances): Occupants of the house have a multiple births or single births bunched

in a two year period This would be a situation where you would want to move out more

quickly than normally Divorce Change in economic circumstances

o However, you do not fall under “unforeseen circumstances” when you simply want to move to another house out of preference

Private letter rulings (PLR) (not quite a powerful of authority as Regs.) Not safe harbors, but things that have met “unforeseen circumstances in private letter

rulings…o A blended family moves to some of the children’s school districto When adult child moves back in with his parentso If you need a bigger house because you have adopted kidso If a disabled parent moves back into houseo Problems with airport noiseo A child was assaulted on a school buso Narcotics officer received death threat from someone who found out where they lived

In short, we are trying to weed out people who are simply moving because they felt like movingo (c)(1)(B): “unforeseen circumstances” calculation

For a single: The shorter of:

o (1): the length of time you were in the house you are sellingo (2): The last time that you used the 121 exclusion

“bears to”o (x/250k) = (n from above/24)o exclusion =

20% tax on all income over exclusion granted **Its a proportionality test to determine how much you can exclude if its been less than 2

years since the last time you excluded the gain  BUT STILL NEED TO HAVE JOB HEALTH OR UNFORESEEN CIRCS

o Otherwise you need to wait two yearso (f): election to not have 121 apply

Sometimes you are better off refraining from applying 12125

Policy effects Employs a transaction basis Congress recognized that an across the board exclusion would inappropriately, from a policy standpoint, give house-

flippers an inappropriate tax breako Congress still wants to reward middle-class homeownership, albeit not to the degree promoted by the old rule

Congress is limiting the benefit to deserving people… “less than 250k for a single and 500k for married”Incentive effects Problem stemming from universal application of numbers no matter where the house

o People were engaging in tax motivated sales as their houses would hit the 250k or 500k marks This occurred in the more expensive, hot markets

o 121 was never amended for regional variation because there is no longer really big gains, let alone big regional disparities in gains due to the decline of any hot markets.

Hypo 1(a) NB lives in Newark; wants to move after 19 months

o Can’t get tax break. Can he get a proportional break due to unforeseen circumstances?

o Instead of getting a 250k exclusion, he is trying to get a proportional exclusion based on 121((c)(1)(B): The shorter of…

(1)o NB moved out of his house in Newark after 19 months.o 19 months

(2)o When was the last time NB used 121? How long has it been since you used the 121

exclusion? He used 121 to exclude a gain on sale while in law school when he sold his

house there. This also was 19 months prior. You might own a house when you move into your new house, then

you might sell the old house after you moved into your new house.o 19 months

“bears to” (x/250k) = (19/24)

o 250k because NB is a single exclusion = 180k

o 20% tax on all income over 180k But does unforeseen circumstances apply?

o Employment? No, he didn’t change schools.o Health? No.o “Unforeseen circumstances” in Regs? No.

He moved because he was bored, so he does not even get the 180k

Hypo 1(b) NB keeps his Ann Arbor house for 12 months while he’s living in Newark

o Only 3k gain on this NB still sells his Newark house still after 19 months

o Gain will be greater on Newark “Shorter of” is now 7 months

o However, if you refrain from applying the 121 exclusion to the sale of the Ann Arbor home, you still get the 19 months to apply in formulating your exclusion from selling the Newark house

Hypo 2 (re: 121(b)(2)(B)) Wilma (W) -- wife

o Lived in and owned A in 2008-2009 Harry (H) -- husband

o Lived in and owned B in 2008-2011

26

W lived in B in 2010 and 2011 W sells A on 1/1/11

o Gain = 350ko W excludes 250k

Pays 20% taxes on 100k remaining H sells B on 1/1/12 due to change in health

o Gain = 400k 121(b)(2)(A)

o Either spouse meets the ownership requirements? Yes.o Both spouses meet the use requirements? Yes.o Is either spouse ineligible due to the recent use of the 121 exclusion? No.

If no 121(b)(2)(A), then 121(b)(2)(B) applies:o Her use of 121 a year ago means they don’t get the whole 500k, so you run H through the three prong mill

(right above) as a single. H meets all three, so you get the full 250k

o W will only get the proportionality rule (the change in H’s health still applies because it his change in health that prompts them to sell the house)

Shorter of… Owned and used…24 months Last use of 121…12 months …calculation

o (x/250k) = (12/24) exclusion = 125k

o together, H and W can exclude 250k +125k = 375k 400k (gain) – 375k (exclusion) = 25k taxable income 20% tax on 25k

Hypo 3 (re: 121(b)(2)(B)):

W lived in (principal residence) and owned her house A in 2009 and 2010 H lived in and owned B from 2009-2012 WH get married January 1 2011, W moves in with H W lived in B in 2011 and 2012  So they have an extra house, A, and they put it up for sale W sells A on July 1, 2011 for $450,000 gain

o Excludes $250k using 121 from 2011 income o Pays 20% of 200k =40k in taxes 

End of 2012, H gets very sick and they need to move into another house without stairs o Need to sell B o H sells B on 1/1/2013 due to health for a 500k gain o How much does he get to exclude?

121 b 2 A i yes-he owned it ii yes-they've both lived there for at least 2 years iii-NO, W a year and a half ago used an exclusion  SO go to OTHER JOINT RETURNS

121 b 2 B Not going to get the full 500k Treat them separately: H fulfills all recs so he gets 250k; W more complicated W meets i, she meets ii because she is deemed to own it because her husband used it SO now onto proportionality test 121 C 2 applies W=x/250k=18/24 (18 months since she sold her home)==>x=187,500

187,500+250,000=437, 500 so taxable gain is 62,500 on which they'd pay 20%

27

DEPRECIATION  Basic idea that property loses value over time through use AKA wear and tear 

Definitely a deduction – you want to correctly measure income Rationale:

This is an income tax, measure income correctly please Correct to undertax

This is a way to reduce tax on businesses...let’s make sure we measure income incorrectly in favor of the business

Overcorrect to undertax even more  Building blocks of depreciation rules

Any depreciation system revolved around the spreading, or allocation, of cost over time and has three elements:

The determination of useful life Taking account of salvage value The application of a method of allocating the cost, in excess of salvage value, over the useful

life  Useful lives

Basic idea is simple, but it is very difficult in practice Section 168

Gives you applicable recovery periods that have nothing to do with reality o Personal Property: basic recovery period

For personal property there are now recovery periods for six different classes of property: 3 year, 5 year, 7 year, 10 year, 15 year, and 20 year

o Real property:  basic recovery periods The recovery periods for real property are 27.5 years for residential

rental property and 39 years for other real property o Intangible assets

Intangible assets such as patents and copyrights are subject to section 167 and are not eligible for the accelerated statutory methods of depreciation described above 

168 is not correct depreciation-its meant to be a tax break to shovel funds back into businesses 

o Policy: Congress subsidizes business because they think itll make the economy stronger

Salvage values Under the MACRS that now applies to all tangible assets, salvage value is disregarded; that is

salvage value is assumed to be zero Favors TP, sacrifices accuracy for simplicity 

Depreciation Method Straight-line

Initial cost -salvage value= x/ divide by useful life in years=annual depreciation amount Not very realistic, most things depreciate quickly at first and then tail off after that

Accelerated Double Declining Balance Depreciation (DDB)

o Determine each years depreciation on a SL method and doubl eit, but then use the remaining value and number of years the follow year 

o So when 35000-0/5000=7000 so you double it the first year and get 14ko Then you act like you only paid 21000 (35-14)-0/4=5,000...etco At some point you realize its better to default back to straightline 

Personal property: basic method The method prescribed for personal property is 200 percent declining balance for 3 5 7

and 10 year property and 150 percent for 15 and 20 property, shifting to straightline when that produces larger deductions 

Real property: basic method The method for real property is straightline 

28

First year applicable convention is that the full year deduction is prorated according to the number of months during which the property is in service during the year

Deduction is prorated by month in the year of disposition Component depreciation is not permitted

Intangible assets In general such assets must be depreciated on a straightline basis 

Year Straight Line Depreciation Double-Declining Balance Adjusted Basis

1 20% * $100,000 = $20,000.00 40% * $100,000 = $40,000.00 $100,000 - $40,000 = $60,000.00

2 25% * $60,000 = $15,000.00 40% * $60,000 = $24,000.00 $60,000 - $24,000 = $36,000.00

3 33.33% * $36,000 = $11,998.8 40% * $36,000 = $14,400.00 $36,000 - $14,400 = $21,600.00

4 50% * $21,600 = $10,800.00 40% * $21,600 = $8,640.00 $21,600 - $10,800 = $10,800.00

5 100% * $10,800 = $10,800.00   $10,800 - $10,800 = ($0.00)

HYPO: How to do double declining depreciation The straight-line method in year 1 determines the percentage of depreciation to be 20% (100% spread over 5

years), which can be doubled for the DDB method, or 40%.  o That means that the taxpayer can use 40% for DDB every year for five years.

The "trick" is that, per section 168(b)(1)(B), at the end of each year, you recompute the possible straight-line depreciation amount based on the number of years remaining (changing the denominator, and thus the straight-line percentage), even though you do NOT change the depreciation percentage for DDB purposes.

For example, after deducting $40,000 in year 1, the remaining adjusted basis is $60,000.  If you switched to straight-line after that year, you'd spread $60,000 over 4 years, or 25% of the remaining basis each year.  This $15,000/year is still less than the $24,000 that is possible using DDB in year 2, so you wouldn't switch yet.  In year 4, however, you'd switch, because the DDB method only allows $8,640, whereas straight-line allows you to deduct $10,800 each year in years 4 and 5, and then you're done.

WHEN IS INCOME TAXED?

ANNUAL ACCOUNTING AND ITS CONSEQUENCES

Rules created by Congress over the years to get you out of arbitrary situations

Sanford and NOLs

Sanford (US SC 1931) Dredging case Income tax system uses annual accounting rather than transactional accounting

o Court gets the law right even though the result is unfair; Congress always adopts laws with annual accounting so they knew what they were doing, both before and after the 16th Amendment

The reason annual accounting produced such harsh results in this case is that losses do not result in negative taxes 

Congress says no we're not going to give you money for maxing out the generous depreciation rules we already gave you!

o Transactional accounting would require distribution of overhead, etc—too difficult Big policy argument

o Any government is going to have to collect income on a regular basis in order to meet revenue requirements

Net Operating Loss (NOL) carryovers Cured Sanford problem stemming from the unduly drastic consequences of taxing income strictly on an annual basis

29

o The harshness of annual accounting is mitigated by the provision in 172 for new operating loss carryoverso The losses that  may be carried over are primarily losses incurred in a trade or business (1212 is for capital

losses) Operating losses are different than capital losses, where you have an asset that loses money over time

o Operating losses=Expenses>Revenues Essentially a business provision Allows you to use extra losses to offset income in other years

o NOL can be carried 2 years back, 20 years forward If you calculate a loss, your tax goes to 0 but we’ll let you hold on to that loss to offset gains in later

years  You continue to offset your gains until you offset all of your losses

o ***If you don't use it in 20 years it goes away  §172: net operating loss deduction

o (b) (2): the loss must be carried back to the earliest year possible… start as far back as possible and

work your way forward (3): allows choice to only go forward and not carryback (but then you can’t carryback for any

year)o (d)

(3): when computing NOL deduction you cannot include deductions from personal exemptions (4): non-corporation…when computing NOL deduction, then you cannot include non-business

deductions “trade or business” never defined by the Code

Hypo 1:o 2009: 40k taxable income [YEAR 0=2010]o 2010: -41k TI

60k salary -80k business loss -6k personal exemptions

o (d)(3) knocks out the 6k -15k nonbusiness itemized deductions

o (d)(4) knocks out the 15k No tax liability The business loss takes him into the negative (and thus the zero bracket) for taxable income He would like to carryback or carryover

He can only carry the 20k of the 41k because 172 disallows carrying the 6k or 15ko However, he can use the 6k and 15k in 2010

However, they do him no good cause he is already in the negative (way in the zero bracket)

The 21k in deductions is essentially gone So, only 20k is his NOL for 2010

o What is his strategy? Allowed strategy

(1) You can go back to year -2 then follow suit, ORo -2…-1…+1…+2…+20

special circumstances allow for a three year carryback (2) You can opt not to carryback to year -1 or -2, and start at year +1

o +1…+2…+20 Cannot just carry to year -1, unless you have nothing to offset in year -2

Here, Assume in 2008, he had nothing to offset

o He can go to 2009 and subtract 20k from the 40k and create only 20k in TI for 2009, OR

o He can apply it to 2011, etc.

30

Hypo 2: (see page 132)o 2009 Company has net loss of 30 ko 2010   30k net taxable income

Does Co. pay tax on 30k income in 2010? NO See 172 b 1 A ii, c 

o No earlier year to carry back to because Corp didnt exist until 2009 o Does this change if 2010 30k net income were attributable to sales of other companies, rather than to profits

on the govt k? NO its NOT transactional accounting...its ALL revenues and ALL expenses

Hypo 3: o Bob had 2012 salary of 60ko Deduction for personal exemptions for 4 k, and itemized nonbusiness deductions of 16k so his taxable income

in 2012 was 40k o In 2013 Bob again earned 60k but suffered an 80k loss from his wholly owned business 

Personal deduction 6k Itemized 15k SO loss of 41k 

o How much income can Bob carry back against his 2012 income  Bob had no income in 2010 or 2011 to carry back to  He can carry back 40k? NO he can't use all of it WHY

172 c d 3 and 4: Can't carry back personal exemptions, can't  use those across years-Congress OR your itemized non business deductions

SO no 6k OR 15k, can only use the 80k (ACTUAL NOL), offset by income, to carry back So he can carry back 20k, and reduces 2010 taxable income to 20k

o **IRC never defines trade or business, uses common law definition instead Hypo 4:

o 2008   2009   2010     2011  2012   2013o 400     700     -3k       700     700    600o    0        0         0        0        0       100o (2600) (1900) (1900)  (1200) (500)  -----o How is the 3k NOL allocated to the other year

Disregarding any special elections...see 172 b 3

Claim of right

North American Oil (US SC 1932)

Facts: There's a lease between NAO and the feds and the feds oust them because NAO did something to upset them They dont have them stop operations they say until we sort this all out we will have a receiver run the

company instead of NAO and they get the oil, sell it, and hold on to the money depending on the results of the ousting case

o Court holds for NAO and they get 172k, which was earned in 1916 Lawsuit not resolved until 1917, then NAO gets paid Gov't doesn't give up though and they appeal, finally give up in 1918

o NAO files an amended tax return to have money count as income in 1916, oh and by the way it happens to have the lowest tax rate

SC holds that it’s def income, just a matter of what year its attributable to so they create the claim of right doctrine: You must include funds as income when you have:

o (1) legal right to the fundso (2) received the funds without restriction

Illinois Power (7th Cir. 1986) The TP, an electric company, was ordered by the IL Commerce Commission to raise its rates

31

Commission made it clear that its purpose was to discourage the consumption of electricity and that the TP would not be allowed to keep the extra revenue resulting from the increase or any interest on it 

IRS argued the money was taxable under North American Oil, but the court rejected this and likened the TP to a custodian with not greater beneficial interest in the revenues collected than a bank has in the money deposited in it 

Custodian has no COR and thus does not have to pay taxes on the money they are holdingo However, what about the interest earned on that money?

Lewis (US SC 1951) In 1944, TP gets a 22k bonus (high tax rates)  In 1946, TP has to send 11k back to the employer (low tax rates)

o No exception to the claim of right doctrine just because the taxpayer is mistaken about the validity of his claim to the money (taxes paid on mistakenly large bonus which he gave back but which he had used completely as his own)

o Income in earlier years, loss in later years, court says tough luck, gotta deduct from later year, when your claim over the money evaporated (DD wanted to deduct from earlier yr, bc higher taxes, and more income then)

Amended returns used to claim a refund of overpayments but only about facts that were or should have been known before end of earlier year AND amendment is within 3 year SOL period

But outcome of this case really is truly unjust, just like S and B When you have arbitrary lines that’s what happens So the Court says let Congress fix it

o Which they do with section 1341 Congress is overcorrecting the Lewis case Let's people like Lewis game play based on what's best for them 

1341: computation of tax where TP restores substantial amount held under claim of right income in previous years that is later lost Congressional overgenerous response to inequity in Lewis…situations involving people who find themselves

in different marginal rate tax brackets in different table years Start with a deduction in the year of repayment (rather than reopening the earlier year)

o Two prong rule:o But, if deduction exceeds 3K, then the tax is the lesser of

The amount determined by claiming a deduction in the ordinary manner, OR By forgoing the deduction and claiming a credit in the year of repayment for the tax that

would have been saved by excluding the item in the earlier year Essentially allows the taxpayer to reopen the earlier year except for the interest on the

overpayment o In essence, the TP may strategically decide what outcome is better for him, rather than forcing

amendment in the year where funds were included in the income Depends on what year’s rates are higher…that’s when you want it

§1341 applies ONLY to those items to which it appeared that the taxpayer had an unrestricted right in one year and to which a repayment is made in a later year because it was established after the close of the earlier year that the taxpayer did not have unrestricted right to the funds

§1341 does NOT apply to restorations based on mere errors or on subsequent events (refunds pursuant to K rights), or to repayment by an embezzler of stolen funds

Tax benefit rule Situation: wrongly taken deduction that you may have to later include (opposite of Lewis—deduction in earlier

year and income in later year)o Took a deduction for a perfectly legitimate reason under your understanding of the facts at the timeo Later, you discovered the facts were not as you thought and you were not eligible for the deduction

§111: recovery of tax benefit items If (or to the extent that) a deduction did not reduce the TP’s tax liability for any year AND any loss carryovers

resulting from it have expired without being used, the recovery of the amount deducted need not be included in 32

income. (i.e., if you are already in the zero bracket, the deduction you wrongly took did you no good…really if it didn’t shift your brackets in any way it did you no good)

o But, if you got any benefit, then you have to include it as income If valuation is difficult you can include it to the extent you deducted it, even though this ignores

inflation Safe harbor: the exact dollar amount you wrongly deducted in the first place is what you

later include Plain English: Deduction did me no good in earlier year, let’s say because you were in 0 bracket, but then I have to

include it in later year, then you don't have to include it BUT it’s all or nothing...even if you only had a tax benefit of $1 then you have to report it all in the

later year Exclusionary Aspect Example:

o Earlier deduction 10ko TP was 6k away from ZB so TI=0o If 10k is restored to you later, you only have to include the 6k because it’s the only amount that helped you in

the earlier year Inclusionary Aspect of the Tax Benefit Rule :

o Arises when the TP has indeed received a tax benefit from a deduction and the includability or amount of the subsequent offsetting gain is not otherwise clear cut 

o The rule requires that income in the amount of the prior deduction be included, thus moving toward transactional rather than strict annual accounting

o Most difficult cases are where the event that matches the earlier deduction is not a recovery of property in the most direct and literal sense  

Alice case: The valuation problem Lots of years have passed so we don't really know how much this unique land is worth any

more, then just used the amount you used for your deduction in earlier year Basically a safe harbor---unless you have evidence its worth a lot less you'd use this

Summary of rules created by Congress over the years to get you out of arbitrary situations

Claim of right The way you choose the year in which you have to include income is on the basis of the two prong test

NOLs: Loss in earlier year, income in later year You have losses in one year that aren’t helpful in one year and you want to carry them over to other years

o Fixing Sanford problem, 2 years back, 20 years forward Even with NOLs under 172, there is no transactional accounting

Tax benefit rule How to deal with a situation in which you had deduction first and income later

o Tax Benefit Rule: treating NOL's as tax benefits TP includes restore amounts to the extent that the deduction reduced taxes

o Inclusionary Aspect: safe-harbor valuation, simple offset (use earlier number) 

1341 How to deal with a situation in which you had income first and lost it later 1341 is an overgenerous response to Lewis

REALIZATION AND RECOGNITION

Timing issueo A taxpayer has income, can they delay paying taxes on that income?

Even if you lost on the merits, you may be able to argue it’s not taxable yet. Why do people care so much about timing?

33

There are several reasons for wanting to defer payment of your taxeso (1) Taxpayer is hoping for a change in rate

In this way, it is a gamble that Congress will lower the tax rates So this is a good reason to use doctrines like realization and non-recognition

o (2) Procrastination a real, albeit not highly defensible, reason

o (3) They do not have the money right now They are either illiquid or waiting on the cash to come in

o (4) As a financial matter, it is valuable to put off paying taxes as long as possible Even if you don’t think rates will change, you’re not a procrastinator, or you’re liquid… Time value of money

If you owe a certain amount in the future, planning to pay that debt through the application of interest rates from putting it in a CD, etc., will allow you

Exampleo Tax bill = $105

How valuable is it for me to pay that tax bill a year from now rather than right now?

o Interest rate = 5%o $100 today (present value) in a bank at a 5% $105 (future value)

by not waiting the year, you are costing yourself $5 You can generalize this concept to any number of rates and any number of interest

rates Rule of 72

o Present value doubles in 72/r years r = 4%, then 72/4 = 18 years

Your present value will double with the interest rate at 4% in 18 years

Because the federal government does not charge interest on legal deferrals o So, every year you defer could earn some moneyo However, interest is charged on illegal deferrals

So, how and why do we allow deferral? Because if it’s beneficial to the taxpayer, it is harmful to the government…

Realization doctrine You do not have to pay taxes on HS income until you have realized the gain

o Realization usually means the sale or disposition of an asset, roughly speaking With a ton of exceptions

Why do we have a realization doctrine a part of our tax Code? (p.225, n.9)o Only when you sell property is the increased value realized!!o (1) Valuation issues

A lot of property that would be subject to taxation under HS income would be very hard to valueo (2) Liquidity problems

You might have the money, but it is not in a form that you can easily cash out o (3) Divisibility of assets

You can’t just sell the fraction of a house to foot your tax billo (4) Variability

If we had a really rigid HS system, then every year you would have people doing this valuation, and frankly we have better things to do with our lives

Unrealized gains in one year may turn into unrealized losses in another year

Recognition doctrine Some situations in which you have income, and have even realized income, Congress gives you the opportunity

to opt not to recognize income and not pay taxes [AKA defer even longer] Why do we have a recognition doctrine a part of our tax Code? (p.247)

34

o (1) Liquidity problemso (2) Valuation issueso (3) Nature of the investment has not changed

gain may be realized but you took the gain and plowed it right back into something that looks identical

o (4) This is a way for Congress to give people money aka CAPITAL MOBILITY Congress can pass recognition rules that amount to tax breaks, but which don’t look enough like tax

breaks to get on the news

REALIZATION

Legal origins of the realization doctrine

Macomber (US SC 1920) Facts: Macomber is wealthy and owns a whole lot of shares in Standard Oil CA

2,200 shares of So Cal, par value 100/share, FMV=800k 1916: SOCAL issues stock dividend for internal accounting reasons, essentially a 3:2 "split" Now 3300 share of SoCal, par value 100/share, FMV=800k

1. IRS says income=3300*100-2200*100=$110k         US SC held that stock dividends are not to be treated as realization events

o Gov’t: Stock dividend is income Court: par value is not how to measure income, not H-S income For Court to reject Congress' def of income (see below) they have to say that the law is

unconstitutionalo Gov’t: Stock dividend is a realization event (equivalent to a sale)

Clear that her stocks have gone up in value in her hands and she has never paid tax on the this increase of net worth

So there is untaxed income there, gov't says this is realized when the dividend is issued Court blows past this argument

Brandeis, dissent: Stock dividend is essentially selling and repurchase SO IT IS A REALIZATION 

The law at the time of this case would have allowed this argument even though it’s stupido Gov’t: increases in wealth can be taxed at any time 

No legal realization theory at this time!! So under current law again, they could have taxed Constitutional argument from majority

o 16th amendment allows taxes on income but we don't know what income is-goodo Need to regard substance over form-goodo Then they go south...

Court says because it’s in stock form and not in cash that proves it’s not income WHICH IS NOT TRUE...income includes more than cash 

o **Immediate holding correct under current law but constitutionality argument no longer good law Brandeis’ dissent points out loophole where form was elevated over substance and the tax outcome is based on

whether it’s called a pure dividend (not taxed) or if SH is given option to buy (dividend is taxed)o Would have been realization if corporation paid M to extinguish shares on the condition that she used the

money to buy new shares

305: modern rules on taxing stock dividends (a): except as otherwise provided, gross income does not include the amount of any distribution of the stock of a

corporation made by such corporation to its shareholders with respect to its stock (b)(1): stock dividend is taxable IF the shareholder has the option to take cash or other property in lieu of that

dividend 1.307-1(a): taxpayer’s total basis of the old shares is allocated between old and new shares based on FMV after

distribution of stock dividend

35

The decline of realization as a constitutional requirement

Bruun (US SC 1940) narrowed Macomber to its facts without overruling Facts:

o 1915 lease startso 1929 new building errected o building had FMV $64k, which replaced another building worth about $12k, so net FMV +50ko bad timing, right before depression o 1933 lease cancelledo 1979 building useful life supposed to endo 2014 lease supposed to endo Bruun ends up paying taxes in 1933

SC held that there is indeed a realization requiremento SC held that the cancellation of a lease because of the default of the renter is one of those non-sale

realization events Realization does NOT = liquidation (non-sales can be realization events)

Realization is not limited to cash exchanges Bruun had to pay taxes on the increase in value of the property from the new building that the leaseholder built on his

land before he defaulted on the leaseo Is Bruun a better case for realization doctrine than Macomber? Yes he doesn't have any cash freed up by this

to pay tax bill o But let's say he has to sell his property to get cash to pay tax, is that so bad? He doesn't use this property; he

didn't want it again until 2014

§109/1019: Bruun statutory aftermath Non-recognition provisions

o You have income, but you don’t have to call it income this yearo Congress doesn't like what happened to Bruun so they pass section 109

Basically just Bruun facts...now he wins and doesn't have to pay tax 109: gross income does not include income (other than rent) derived by a lessor of real property on the

termination of a lease, representing the value of such property attributable to building erected or other improvements made by the lessee

o You can temporarily exclude the value of a building at the end of a leaseo But, to prevent abuse, you can NOT exclude value of property if building is in place of rent

1019: Neither basis nor adjusted basis will be changed based on income generated for lessor that was not taxed due to 109

109/1019: when the lease gets cancelled, technically it’s still a realization event, but you don’t have to recognize it.o Still income, but you just get to put it off, until you choose to sell the land, if you ever choose to sell it.

You will pay tax on the same amount of money at some point But clearly you’d rather be in the tax-friendly realm of 109/1019

o While this isn’t the kind of giveaway we’ve seen before, the taxpayer can surely earn interest by delaying paying taxes

1.61-8(c)o No safe harbors; no specificityo But, makes the rule clear

A building intended as rent has to be treated as a perfect substitute for the FMV of the annual rent, thus the realization event cannot be delayed as in the 109/1019 situation above

Applying this in real life situations will be really difficulto This is the better way to handle it, but this isn’t how we handle it. We use

109/1019. We act as if the building is not yet in the possession of the lessor and thus he

pays no income taxes on it, but also gets no deductions

36

Hypo 1 (p.26 (Notes)), P. 212 BOOK Facts

o Building = 50k 10 years of life

o Annual rent = 7k per year Bruun treatment

o 1933: 50k income Gives Bruun 50k in basis (important if he goes to sell it)

If Bruun sold it a minute later for 50k, he would have no income on which to pay taxeso 1933-1943: 7k per year in rent

Building is losing 5k in value per year – depreciation Theoretically, a 1/10th of the building crumbles each year

So, he has 2k per year in income that is taxable 20k over ten years

o A total of 70k (50k + 20k) is included in his income from 1933-1943 109/1019 treatment

o 1933: 0 income gives Bruun 0 in basis in the new building (important if he goes to sell it)

o 1933-1943: 7k per year in rent Because Bruun didn’t pay taxes on the 50k when he received it, he doesn’t get deductions on the

building’s decay Bruun did not buy the building up front

So, he has 7k per year in income that is taxable 70k over ten years

o A total of 70k is included in his income from 1933-1943 Bruun will pay tax on $70k at some point

But clearly you’d rather be in the tax-friendly realm of 109/1019o While this isn’t the kind of giveaway we’ve seen before, the taxpayer can surely earn

interest by delaying paying taxesHypo 2 (p.63-64 (Notes)) You are the owner of the land Person renting the land puts a 400k building on it

o HS income…400k increase in net wortho Other possibilities…

Wait until year 10 The building is half used up and worth only 200k Include 200k in income and take your depreciation deductions for every year after that

o Acting as if the building is not in your possession for the first 10 years Here, we are pro-taxpayer a la 109/101

The limits of realization: nonrecourse borrowing in excess of basis

It is now widely understood that any sale or other deposition of an asset will be regarded as a realization event, thus requiring the TP (in the absence of an applicable non-recognition provision) to report any gain or loss from the sale on her fed income tax return

Recourse v. non-recourse loanso Recourse

Borrower is personally liableo Non-recourse

Lender can only get whatever is secured Lender has no access to borrower’s personal assets

Woodsam (2nd Cir. 1952) Facts: Mrs. Woodsam had bought the property in 1922 for 296k

37

o Basically got a bunch of mortgages and kept consolidating them until she got a 400k mortgage that was nonrecourse (aka she was not personally liable)

So how do you treat the 381k mortgage balance when the building get foreclosed in 1943?o Example of a non-realization event (new kind of loan)

Court held: Foreclosure is a disposition event and gain/loss would have to be realized then, however the change in the type of loan (from recourse to nonrecourse) is not a disposition of the property

o Despite change in loan terms, taxpayer has control of building therefore it has not been not disposed of, therefore not realized

o Foreclosure may result in DOI income, but 108(b) has exemption if D insolvent **Very narrow holding based on complicated facts

Contemporary understandings of the realization doctrine

SL crisis: The response of regulators to the SL crisis was to allow SLs to go ahead and gamble in the hopes that they would get

through the crisis until the interest rates come back down and the SLs become solvent againo Dramatic increase in interest rates during the 1970so Reduced the value of existing fixed-rate mortgages held by thrift institutionso Fair market value of mortgages by the nation's savings and loan institutions had declined by over a trillion

dollars  Recognizing these losses for tax purposes would generate valuable loss deductions for the thrifts, but

an outright sale of the mortgages was off the table for accounting reasons To have their cake and eat it too, the thrifts would have to find a way to sell their mortgages for tax

purposes without selling their mortgages for accounting purposes Problem: maturity mismatch 

o Now have to charge more on mortgages to pay more interest on depositso But the mortgage are all maturing can't start charging more interest  yet until you get

new mortgages So now those are skyrocketing

Latest recession: how was this different than S and L crisis 25 years earlier? o Banking structure changed, no more wall between investment banks and deposit institutionso Financial institutions investing in boring mortgages, but regulations allowed them to invest in risky

mortgages o Mortgage-backed assets then began to fail; everyone was invested in same undiversified portfolio, system

unraveled bailout occurred 

Cottage Savings ***NB thinks the majority decision is awful in every respect; the dissent gets it right Facts: FHLBB responded to S and L crisis by relaxing its requirements for the reporting of losses: memo R-49

determined that S and L need not report losses associates with mortgages that are exchanged for substantially identical mortgages held by other lenders. P sold 90% participation in 252 mortgages to four S and Ls and simultaneously purchased 90% participation interests in 205 mortgages held by these S and Ls 

o On its 1990 return, P claimed a reduction for 2.4 million which represented the adjusted difference between the FV and the FMV

o As permitted by the memo, it did not report these losses to FHLBB  SC

o Acknowledge that there was no sale…but was there a realizationo Whether there is a realized loss depends on a two-prong analysis

(1) There is “material difference” requirement built into the realization doctrine. 1001. 1.1001-1: “differing materially in kind or in extent”

(2) “Material difference” Distinct legal entitlements make an exchange of properties materially different

o Essentially the Court adopts the taxpayer’s argument that any difference is a material difference

38

properties may very well be substantially identical, but materially different

properties are different in the sense that is material to the IRC so long as their respective possessors enjoy legal entitlements that are different in kind or extent 

Here, because P's participation interests exchanges were made to different obligors and secured by different homes, the exchanged interests did embody legally distinct entitlements 

Dissent (Blackmon) (Prof: DISSENT IS RIGHT)o Majority applied form over substance

What's the harm of CS? o Court wanted to call it a realization event so they said if theres ANY distinction between the two things

exchanged, then you get a realizationo But ppl who are exchanging things that have GAINS they dont want a hair trigger requiremento IRS also wants a predictable rule and doesn't want to deal with hair trigger rule

Standard going forward: Establishment of a “hair-trigger” standard… any difference = realization event

o Any difference at all, under CS, is going to lead to a disposition and therefore a realization event Ever since CS, the regulators have been trying to establish a “materiality” requirement

The Regs have tried to reduce the hair-trigger standard produced by CSo Put materiality back into materially different –“significant modification”

o Any refinancing would be a realization event

Applying CSHypo 1: J and B

o Cotton dealerso Business acquaintances

Both have cotton in a warehouse Price of cotton has fallen

o So both have substantial unrealized losses on the cotton they have in storage Both would like to avoid selling, but both have large gains and would like to avoid losses 2 ways of running the transaction

o Leaving your bale of cotton in a warehouse for storage and getting a bail back This is the non-tax, most sensible way to store cotton

o Leaving your exact bail of high-grade cotton in a warehouse for storage and getting an equally high-grade bail, albeit not the exact same one, back

Could report losses here under CS Under CS, whether there is a loss is YES but under IRS regs after CS the answer is NO

o Under CS if they got a warehouse receipt that same you could come get YOUR bails o So you have to separate the cotton in the warehouse...wasteful 

Hypo 2: S, in NYC, has the legal entitlement to Streaker 13 and J, in Miami, has the entitlement to Streaker 14

o Cars are sent to the wrong city; each dealer has the docs establishing ownership Both cars have 750k FMVs, both have a 500k basis (What they paid) The sensible thing to do would be to send the papers through the mail

o But, the papers are distinct legal entitlements And because they are distinct legal entitlements, the exchange of them is a realization event

The trade of papers is a 250k realized gain for both S and J

39

o If they did the sensible thing, it would weirdly trip to CS rule and provide both taxpayers with a gain that they don’t want

S and J may very well go through the ridiculously wasteful process of shipping the cars rather than the paper We apply notions of symmetry here

o CS triggers a realization event in situations that aren’t beneficial for the taxpayers

NONRECOGNITION

Introduction to nonrecognition rules

Section 1001c: Except as otherwise provided in this subtitle, the entire amount o the gain or loss, determined under this section, on the sale or exchange of property shall be recognized 

In most cases, this language ensures an automatic connection between realization and recognition ; TP who realizes a gain or loss from the disposition of property must recognize such gain or loss for tax purposes 

Except as otherwise=nonrecognitiono Chief function of a non-recognition rule is to ensure that a TP disposing of property in a manner that would

otherwise be a taxable event will not recognize gain or loss on the disposition of the property  Not a permanent exclusions but a deferral to a later time 

Rationale  o Gain should not be recognized if the transaction does not generate cash with which to pay the tax (admin)o Gain or loss should not be recognized if the transaction is one in which the gain or loss is or might be difficult

to measure (admin)o Gain or loss should not be recognized if the nature of the TPs investment does not significantly change

(fairness)o Gain should not be recognized in order to avoid discouraging mobility of capital (econ)

Underlying incentives stemming from the realization doctrineo If people have lost money on assets, the realization doctrine gives people with losses the ability to realize

those losseso On the other hand, if you have gains, you want to hold off paying gains as long as you can by avoiding

realizing them If you can hold off on realizing gains until you die, then you have completely avoided paying

taxes on those gains Congress doesn’t like some the incentives in some of the situations created by having a realization regime…

o So, they created a nonrecognition rule

Realization v. Recognition

Realization: You exchanged your property for cash or some other property...and is it materially different? Non-regconition: Congress says some of the realization events are going to get special treatment

o Nonrecognition rules are “giveaways” Defers when a person has to pay taxes—valuable!…think about the time-value of money

Hypos on p 229:o Yes, taxable (swap exception)o No, 1031o Yes, happens the next week, can't cash out and buy back in under 1031o Yes, tractors (farm equipment) and farms are not like kind

Like-kind exchanges Most prominent non-recognition rule: 1031, applies to exchanges of certain business or investment property that are

held to be of a like kind §1031: exchange of property held for productive use or investment Like kind exchanges are granted non-recognition status (even when substantially different)… it IS income, and

it IS realized, but congress exempted it from recognition

40

o (a): no gain or loss is recognized upon exchange of property held for productive use in trade or business or for investment solely for property of a like kind to be held either for productive use in trade or business.

Excludes: inventory held for sale Financial assets easy to value and liquidate

o E.g., stock, bonds, notes, securities Partnership interests Certificates of trust/beneficial interests Choses in action

Policy o This is supposed to be a business-enhancing rule

1.1031(a)-1o LK means two things that are darn close to the same thing

LK is much broader than “substantially identical”o Focus is on the nature and character of the property and not on its grade or quality

Private Letting Ruling: Therefore, TPs exchange of a PCE in real property, for fee interest in other real estate that is also subject to a PCE will qualify as a tax deferred exchange of like kind property, provided that the properties are held for productive use in a trade or business or for investment 

1.1031(a)-2o Creation of classes

If you are in a like class, means you are LK If you are not in a like class, you might be LK

o General Asset Classes (GAC) Fairly arbitrary distinctions created by Regs. Writers

Even though arbitrary, they act to provide guidance to those seeking it Hypos in book 232-233

o Exchanging computers for printers are OK because both in same asset class o Buses for airplanes....not in same asset class..but are they are like kind still?

Good business judgment about using it for same general purpose of transporting people

Boot and Basis

Boot o The extra money/property need to even out an exchange deal=booto Transfer of boot will affect basis and may result in the recognition of gain 

Basis o Basis is correctly keeping track of how much you paid for the asset so that you can realize the proper

gain/losso Section 1031 d sets forth the rule for determining the basis of property received in an exchange OF LIKE

KIND and nothing else Generally there will be a substituted basis, that is the basis for the property received will be the same

as the basis of the property relinquishedo Rule becomes slightly more complicated when there is boot

If you have an exchange with boot, you only recognize the gain up to the amount of the boot You recognize the lesser of the gain realized or the total value of the boot

When you have a loss you have  no recognized loss Which makes sense because you should just sell the thing instead of exchanging it so you can

recognize the whole loss 1031(a): pure LK exchange

o You are exchanging only LK things…non-recognition ruleo Provides a safe harbor if exchanged items in same asset class (13, defined p. 630 SUPP)

1031(b): mixed LK exchange

41

o If you give up a piece of LK property and receive back LK property and non-LK property and that is a gain to you, then you will have to pay tax on some of the gain, but not necessarily all of it depending on how much you got back was boot

Mixed LK exchange…some of the gain is recognizedo Shorthand

You will have to recognize the lesser of the gain or boot 1031(c): mixed LK exchange

o Mixed LK exchange…none of the loss is recognized Asymmetry between (b) and (c)

o Shorthand Losses are not recognized

But losses are easy to take advantage of because you can sell the thing rather than engaging in a LK exchange

Mixed LK exchange summary o Recognize the lesser of the gain or boot

However, losses are not recognizedo Formula (Calculations on p.33 (Notes))

Proceeds of the disposition Proceeds – Basis you had in property disposed = gain realized Recognized gain

Lesser of o boot received in proceeds, OR o gain realized

If gain realized is negative, do not recognize losses 1031(d): what you do with the basis after applying (b), and (c)…mixed LK exchange

o Formula (Calculations on p.33-34) A (original basis) + B (gain recognized) = C (total basis) – D (FMV of the boot) = E (new basis of

the like-kind property received)…useful equation1. First compute realized gain!!2. A + B = C3. C – D = E

a. A (original basis) + B (gain recognized) = C (total basis)b. C (total basis) – D (FMV of boot received) = E (new basis of the LK property received)

Total basis (C)o If you’ve recognized the gain, then you paid taxes on it, and you are done paying

taxes on that income. So, we insert the gain into the basis so that you can later deduct that amount when you sell

You want to figure out the new basis because you won’t hold onto the new property forever and you’re going to want to know what basis you have in that property

How the equation comes out depends upon whether you are the payor or the payee of the boot Whether an exchange is pure or mixed depends on whether you are the payor or the payee of the boot

Sample complete calculations: mixed LK exchange problems (1) S owns Farm X and has a 10k basis in it; T owns Farm Y

o Exchange: T owns Farm X; S owns Farm Y S also gets 15k in cash and an 8k tractor

So, there is a 23k booto FMVs

Farm Y FMV = 100k, so Farm X FMV = 123k

This is deciphered from the 23k boot (and also assuming that this is an arm’s length transaction

o Boot/Basis Proceeds from the disposition of X

42

100k + 15K +8k = 123k Amount realized = Proceeds – Basis 123k – 10k = 113k

S has a 113k realized gain Recognized gain is the lesser of the realized gain or boot = 23k boot (rather than 113k realized gain)

o New Basis 10k (original basis) + 23 (gain recognized) = 33k (total basis) 33k (total basis) – 23k (FMV of boot) = 10k (new basis of the LK property received)

B = D…E remains the same (substituted basis) Basis when you sell property = 10k

(2) S owns Farm X and has a 110k basis in it; T owns Farm Y o Exchange: T owns Farm X; S owns Farm Y

S also gets 15k in cash and an 8k tractor So, there is a 23k boot

o FMVs Farm Y FMV = 100k, so Farm X FMV = 123k

This is deciphered from the 23k boot (and also assuming that this is an arm’s length transaction

o Boot/Basis Proceeds from the disposition of X

100k + 15K +8k = 123k Amount realized = Proceeds – Basis 123k – 110k = 13k

S has a 13k realized gain Recognized gain is the lesser of the realized gain or boot = 13k realized gain (rather than 23k boot)

o New Basis 110k (original basis) + 13 (gain recognized) = 123k (total basis) 123k (total basis) – 23k (FMV of boot) = 100k (new basis of the LK property received) Basis when you sell property = 100k

The basis you had in the original property was 110o The 10 in boot that you didn’t pay taxes on is made up for by knocking off 10 of

your basis (3) S owns Farm X and has a 130k basis in it; T owns Farm Y

o Exchange: T owns Farm X; S owns Farm Y S also gets 15k in cash and an 8k tractor

So, there is a 23k booto FMVs

Farm Y FMV = 100k, so Farm X FMV = 123k

This is deciphered from the 23k boot (and also assuming that this is an arm’s length transaction

o Boot/Basis Proceeds from the disposition of X

100k + 15K +8k = 123k Amount realized = Proceeds – Basis 123k – 130k = -7k

S has a -7k realized gain…no recognized gain because it is a losso New Basis

130k (original basis) + 0 (gain recognized) = 130k (total basis) 130k (total basis) – 23k (FMV of boot) = 107k (new basis of the LK property received) Basis when you sell property = 107k

The rule says you have 7k loss that 1031 prohibits you from reaping a tax advantageo When you sell your property later you subtract 107k instead of 100k…there is your

7k loss43

(4) Example of when you are giving up boot, rather than receiving booto T owns Farm Y (100k FMV)

T has 10k basis in Farm Yo T exchanges Farm Y + 15k cash for Farm Xo Boot/Basis

Pure LK exchange for the person giving up boot…so no recognized gaino Basis

10k (original basis) + 0 (gain recognized) = 10k (total basis) 10k (total basis) – (-15k) (FMV of boot) = 25k (new basis of the LK property received) Basis when you sell property = 25 k

(5) Exam review hypo Formula

o A (original basis) + B (gain recognized) = C (total basis)o C (total basis) – D (FMV of boot received) = E (new basis of the LK property received)

Factso Y owns Greenacre

Basis in GA is 50k FMV of GA is 130k

o Z owns Redacrre Basis in RA is 10k FMV of RA is 200k

So, Z will require booto Exchange

Z gets GA + 70k in boot Y gets RA

o Z’s basis in GA 200k (130k + 70k) (proceeds) – 10k (basis) = 190k

Z has a realized gain of 190k Recognized gain is 70k (lesser of boot or gain)

Thus 70k TI here, and it is paid up front 10k + 70k = 80k – 70k = 10k

Basis = 10ko If she sells GA tomorrow (or in some time where FMV doesn’t change)…

130k – 10k = 120k TIo Y’s basis in RA

(200k – (50k basis in GA + 70k boot)) = 80k Pure LK exchange for the person giving up boot…so no recognized gain

o Thus no TI here 50k + 0 = 50k – (-70k) = 120k

Basis = 120ko If she sells RA tomorrow (or in some time where FMV doesn’t change)…

200k – 120k = 80k TIo Coming full circle

Z is ultimately paying tax on her 190k gain Y is ultimately paying tax on his 80k gain

Timing Review

Realizationo Macomber

Creates realization requirement Stock dividend is not only not realized, it is not even income

o Bruun

44

Non-sales can be realization events Bruun is partially overridden by 109 and 1019 (these help fix, DO NOT overrule—limit to facts)

Particular situation involving lease ending and property coming back to the ownero Woodsam

There are non-cash transactions’s that are NOT realization events, and there are non-cash transactions’s that ARE realization events

o (Cottage Savings) Distinct legal entitlements, In brackets because it has been appropriately (very) narrowed

Nonrecognition o Like-kind

Non-recognized except to extent of booto Private Letter Rulings

Nature or character defines what is LKo Regs.

Give you rules and examples to determine LK, safe harborso Basis equation

Tells you how to adjust basis after a LK exchange

TRANSFERS INCIDENT TO MARRIAGE AND DIVORCE

Introduction

Recognition question:o When a spouse, pursuant to a divorce, transfers wealth to the other spouse, should the transfer be treated as a

realization event for the transferring spouse? Current law re transfers incident to divorce

o Property v. Alimony Property (on-going financial obligations)

Treated like a gift o Property is not treated as income to the recipient and is not treated as a

deduction to the payoro Payee takes basis of payor

Alimony Not treated like a gift

o Alimony is income to the recipient and is a deduction to the payoro Payee does not take basis of payor

Child support Child support is not treated as income to the recipient and is not treated as a deduction to

the payor since money is same as would be paid as a personal expense if living with child and not deductible

Property settlements

Transfers incident to a divorce or separation agreement

Davis (US SC 1952) Facts: As a division in settlement of their property, the TP agreed to transfer to his wife 1,000 shares of stick in the

DuPont companyo Mrs. Davis agreed to accept this division in full settlement and satisfaction of any and all claims and rights

against the husband whatsoever SC’s analysis

o This was a taxable event Mr. Davis’s Gain

45

Taxable gain = Proceeds (Inchoate marital rights = FMV of shares) – Basis in shares held by Mr. Davis

o Inchoate marital rights are equal in value FMV of the shares (re: Proceeds)o We assume that the parties acted at arm’s length and that they judged the marital

rights to be equal in value to the property for which they were exchanged You have to treat it as if she bought the shares with something of equal value

o Otherwise, it becomes problematic Mrs. Davis’s basis in the shares bargained for:

Her basis is the FMV of the shares on the day of divorceo Exchange was not a gift, so basis does not transfer…1041 cures this…

§1041: transfer of property between spouses or incident to divorce Congress’s response to Davis

o It is not a gift, but by legislative fiat, it is given gift treatment Basis is transferred to recipient of property (it is no longer the FMV of the shares on the day of

divorce/transfer) (a) No gain/loss shall be recognized on a transfer of property from an individual to a spouse or former spouse (if the

transfer is incident to the divorce [defined in 1041(c) to mean occurring within 1 year after marriage ceases OR related to the cessation of the marriage]

o 1.1041-1T, A-7: a transfer of property is treated as related to the cessation of the marriage IF the transfer is pursuant to a divorce or separate instrument AND the transfer occurs not more than 6 years after the fate on which such marriage ceases

(b) In this case the transfer is treated as acquired by gift and the basis of the transferee will be the adjusted basis of the transferor

Statute makes the difference between community and separate property irrelevant Recipient of property will have to pay tax on original basis ***ONLY APPLIES TO MARRIED COUPLES (not i.e. same sex couples who separate)

H and W married then get divorced (p 296 notes)

1. Had a house for 400k with basis of 100k 2. Pursuant to the divorce, H took the title of the house and executed a promissory note for 200k payable to Wilma

and secured by the house 1. What amount of gain, if any, is recognized by Wilma? What's W's basis for the note?

1. First, how much was realized 1. They jointly owned the house2. W had half the basis and she sold her half to him 3. W's realized gain=200-50k=150k

2. Recognized gain=0 under 1041 a 23. Basis for the note=$200k....she's selling him the note for 200k

2. H's basis for house=100k=50k (his basis) + 50k (her basis) under 1041 b 2 1. Skews the logic2. Under 1041 if the husband then sells the property for 400k, his gain is 300k (instead of 150k like

below)3. From H's perspective this isn't really a 50 50 split4. You need a lawyer to tell you that 1041 will apply

3. If both H and W have a tax rate of .2 1. 1041: W pays 02. 1041: H pays .2*(400k-100k=300k)=60k3. If you want to advice him well, you need to tell him he's going to pay her tax bill essentially4. Now apply 121

1. No longer married so not filing jointly, he only gets up to 250k exclusion of the gain2. 300-250k=50k*.2=10k

4. Davis: What would be your answer to A and B in the absence of 1041 1. Realized gain would be 150k, so recognized gain would be 150k

46

2. His basis without 1041=50k (his basis)+200k (her basis)=250k3. NO 1041, tax rate=.2

1. W will pay=.2*(200-50k)=30k2. H will pay=.2*(200-50k)=30k

4. Now apply 121 1. Still only gets 250k but gain is 150k so no tax 2. And she would get the same all excluded 

5. They could plan around this if they are amicable3. Question 2

1. 121 d 3 B 1. Divorced six years ago2. 121 says ownership or use within last 5 years 3. Continued to each own half of the house until the kids grow up and then they are ready to sell the

house 4. 121 only available as non-joint buyers because they are divorced

1. For H its easy he's lived there and hes owned it2. For W shes owned it but in the last 6 years she hasn't lived there...so she can't exclude her

half of the gain IF it weren't for 121 d 3 B:1. Individual should be treated as living in the property in this exact scenari-->other

spouse stayed in house to raise kids

Prenuptial settlements

Mercer (2nd Cir. 1947-pre-dates Davis) Facts: Kresge is married (very wealthy, owns K-mart), and 57 (basically dead) 

o 32 year old woman is his newer model, decide to get married but sign a prenup o As part of the prenup he gives her 800k in stock of K-mart aka 11-12 million before marriage

His basis $.15/share, FMV at transfer $10/share, Ultimate sale price $19/share Prenump calls the 800k both a gift and consideration         

Mr. Mercer’s Gain o Taxable gain = Proceeds (Inchoate marital rights = FMV of shares) – Basis in shares held by Mr.

Mercer Mrs. Mercer’s basis in the shares bargained for:

o Her basis is the FMV of the shares on the day of transfer Exchange was not a gift, so basis does not transfer

Congress overruled Davis with 1041, but Congress did not overrule Mercer because the transaction occurred before marriage

o However, you can elect yourself into 1041 by specifying for the transfer of property in the pre-nup to occur after the marriage

Remember: Davis treatment is equal tax treatment, 1041 is not equal tax treatment

Basis in Davis and Mercer Timeline

o 0…1…2….3 0 – husband buys shares

this gives husband his basis 1 – husband marries wife 2 – transfers shares to W; also, divorced 3 (not necessarily one year later) – W sells shares

o Tax-relevant transactions (1) In Y2, H realizes gain on sale of shares

We are looking at it as H sold the shares on the open market and W bought them back TI (Y2) = value of the inchoate marital rights in Y2 – basis of shares in Y0

o Value of inchoate marital rights is presumptively equal to the values of the shares themselves

47

This is less compelling in the Mercer case because divorce is more of an adversarial situation

Pre-nups are not as adversarial and it may very well be that this equivalence is really off

When do inchoate marital rights come into existence? Davis…year 1, when they took the vowels Mercer…not as clear as Davis

o Inchoate marital rights occur even before marriageo W signed away marital rights contractually

So, inchoate marital rights may come into existence before year one, but when exactly?

o If we can’t point to the exact moment, should her basis in transaction (2) be 0?

…TI (Y2) = FMV of the shares in Y2 – basis in Y0 (2) In Y2, W realizes income from selling her inchoate marital rights

TI (Y2) = FMV of shares in Y2 – FMV of shares in Y2o Under 1041, H’s Y0 basis is in the shares that transfer to W

But isn’t this problematic? Shouldn’t the basis be 0 as suggested above? (3) In Y3, W realizes income from stock sale

TI (Y3) = FMV of shares in Y3 – basis in Y2o Under 1041, W gets transferred H’s basis in the share from Y0

(4) … Hypothetically… In Y4, H disposes of inchoate marital rights for 0 TI (Y4) = 0 (Proceeds) – Basis (Y2)

o So is there a tax loss that H never realizeso In summation…this discussion…

Highlights tax logic Congress did a smart thing by saying that inchoate marital rights should not be viewed as

property So, considering that they are not property, we treat inchoate martial rights as gifts

However, you still need to be aware of Mercer If the nexus of the transfer is not tied so closely to marriage, you have to make sure to define

the transfer of the gift …Key to this discussion: as a matter of understanding divorce transactions, this is what 1041

avoids (or gift transactions if not in marriage)

Alimony and child support

Aside from transfers of property, the only other transfers incident to divorce are alimony and child support Again, only applies to marriage, so DOMA is relevant Basic Scheme

o Code establishes a coordinated system in which, following divorce, certain payments received by the payee spouse are taxable to him or her under 71 a and those payments become deductible by the payor spouse under 215

Other payments are not taxable to the payee spouse and are not deductible by the payor spouseo Alimony is taxable to the payee and deductible to the payor, while child support and property

settlements are not taxable to the payee and not deductible by the payor Child support v. alimony

o Which one did he pay? In full? proportional?o Matters because alimony is deductible and child support is noto 71 c 3 is a clumsy way of saying it’s not proportional

You presume the payment is for child support first until the child support is paid in full and then the rest of it goes toward alimony

Alimony

48

215/71/62: the recipient includes alimony in their income and the payor deducts it The requirements (71) to receive alimony treatment

o (1) Payment must be in cash In cannot be stocks, land, a car It can be a check

Cash means not propertyo (2) It has to be a written instrument

If there is a written instrument of divorce, you can presumptively get 215/71 treatment o (3) You can opt-out of 215/71 treatment

However, you can opt out of 215/71 treatment in such an agreement o (4) Parties cannot be living in the same household

Anti-fraud part of the Codeo (5) Payments have to end at the death of the recipient

If they don’t, then apparently they weren’t alimonyo (6) Cannot be payments for child support

Substance over form…i.e. if it terminates when a child is 18 but you call it alimony…clearly not alimony

o (7) Must be substantially the same the first three years [see front loading below] 62: alimony is an above the line deduction

Remarriage Generally alimony payments are deductible to payor and included as income to recipient, but if W remarries…

o RR 82-155 – if recipient remarries and payor is no longer under a legal obligation to pay but does anyways, recipient still has to include it in income if payor does not know that legal obligation no longer exists

o If payor does know that obligation know longer exists but pays anyways, then not included as income by recipient

Child support

71(c): the recipient DOES NOT include alimony in their income and the payor DOES NOT deduct Alimony does not apply The custodial spouse is going to be receiving money and the divorce instrument has to say how much is for alimony

and how much is for child support Child support, unlike alimony, is not included in the income of the custodial spouse

o Child support payments are not taxable to the recipient because the recipient essentially has basis in that money

The recipient is really a pass-through that spends the money on the childreno Not taxable to the recipient because this is not a gain

Not deductible to the payoro The tax logic is that if they had not divorced he would be paying this money to support his children and that

would not be deductible These kinds of expenses are personal expenses

71(c) maintains a structure that is consistent with the rest of the Code **Incentive to lie and call child support alimony

o Often push on that during divorce negotiations 

Examples p 303 Problem 1

o Answer: amount of money involved is under an oral agreement and therefore not part of a divorce decree and therefore not treated as alimony

not income to max, not deductible to W Problem 3

49

o Not alimony because its payable after death; and insurance policies are not deductible because they aren't cash 

Problem 4o child support, non deductible to M and not includable for W

Problem 5o Not cash, therefore not alimony so be treated as transferred basis

Excess front-loading

71(f) Tough tax provision Essentially, this is a very clumsy way for Congress to make sure that substance dominates form

o Remember: There is a tax advantage to be had here (p.323 (Klein)) Most times…wealthier spouse gets the deduction and poorer spouse has to include the sum in income

Poorer spouse is a lower bracket, if not the zero bracket…so, there is a revenue loss to the Treasury

If there are excess alimony payments, the payor spouse shall include the amount of such excess payments in gross income for the payor spouse's taxable year beginning in the 3d post-sep year 

o Means taxable year beginning in third post sep year So if you get divorced in Oct 2013, first taxable year is Jan 1, 2014 So third post sep year starts Jan 1, 2016

Includes months more than 3 years after divorce o But for our purposes we are going to assume that taxable year and post-sep years start the same 

So what does this mean? We took payors word for it that it was alimony in first year when he paid 50ko But two years later there hasn't been alimony payment so we say hmm well then that wasn't really alimony

was ito And then we say okay you can only treat the first 15k as alimony and you have to include the 35k to offset the

50k deduction they got the first yearo Could still hurt payee spouse

Formula (Calculations p.90- (Notes)) o Payments (numbers in ks)

Y1 = ?; Y2 = ?; Y3 = ?o Analysis (71) (13 steps)

(f)(4)(B)(i): (Y3 payment) (f)(4)(B)(ii): 15k (f)(4)(B): (Y3 payment) + 15k = (f)(4)(B) (f)(4)(A): (Y2 payment) (f)(4): (Y2 payment) – (f)(4)(B) = (excess payment for Y2)

negative number = no excess payment for Y2 (f)(3)(B)(i)(I): (Y2 payment) –(excess payment for Y2) = (f)(3)(B)(i)(I) (f)(3)(B)(i)(II): (Y3 payment) (f)(3)(B)(i): [(f)(3)(B)(i)(I) + (Y3 payment)]/2 = AVERAGE (f)(3)(B)(ii): (average from above) + 15 = (f)(3)(B) (f)(3)(A): (Y1 payment) (f)(3): (Y1 payment) – (f)(3)(B) = (excess payment for Y1)

negative number = no excess payment for Y1 (f)(2): (excess payment for Y1) + (excess payment for Y2) = (f)(2) (f)(1): In Y3, payor includes (f)(2), recipient deducts (f)(2)

The point of all of this is curing the one-time payment termed alimony, but which in effect wasn’t alimony because alimony is to be received over time

o Recapture

Exam review hypo

50

Factso Y1 = 75; Y2 = 35; Y3 = 10

Analysiso (f)(4)(B)(i): 10 (Y3 payment)o (f)(4)(B)(ii): 15o (f)(4)(B): 10 (Y3 payment) + 15 = 25 (f)(4)(B) o (f)(4)(A): 35 (Y2 payment)o (f)(4): 35 (Y2 payment) – 25 (f)(4)(B) = 10 (excess payment for Y2)o (f)(3)(B)(i)(I): 35 (Y2 payment) –10 (excess payment for Y2) = 25 (f)(3)(B)(i)(I)o (f)(3)(B)(i)(II): 10 (Y3 payment)o (f)(3)(B)(i): [25 (f)(3)(B)(i)(I) + 10 (Y3 payment)]/2 = 17.5 AVERAGEo (f)(3)(B)(ii): 17.5 (average from above) + 15 = 32.5 (f)(3)(B)o (f)(3)(A): 75 (Y1 payment)o (f)(3): 75 (Y1 payment) – 32.5 (f)(3)(B) = 42.5 (excess payment for Y1)o (f)(2): 42.5 (excess payment for Y1) + 10 (excess payment for Y2) = 52.5 (f)(2)o (f)(1): In Y3, payor includes 52.5 (f)(2), recipient deducts 52.5 (f)(2)

Hypo p 302o Alimony 50k in first year and no payments in years 2 and 3 

Why? its deductible if they call it alimony o Does this violate the excess front loading rules?

Clearly it does o 71 f 4 B

0+ 15k=15k 4A=0 4: 0-15k=0 (because it would be a negative number)

o 71 f 3 3 B i I: 0-0=0 3 B i II: 0 Average of two numbers under i is 0

o 3B: 0+ 15k=15ko 3A: 50ko 3: excess if any of 50k-15k=35ko 71 f 2: sum of excess payments of first post sep years (35k) and excess payments for 2nd post sep  year

(0k)=35k Another hypo

o 50 k, 30k, 10ko 71 f 4 B: 10 +15=25ko 4A: 30ko 4: 30k-25k=5ko 3 B i I: 30k-5k=25ko 3 B i II: 10ko B i: 25+10k=35k/2=17.5ko B: 17.5k+15k=32.5ko 4 f 3 A: 50ko 3: 50-32.5=17.5ko 2: 17.5 +5k=22.5k payor spouse has to add back into income in year 3 and payee can deduct it

***Not going to ask us to do ALL of the calculations, but he might say assume you know X part of the equation...what's the next move with language from 71?

1. Alimony paid in 2nd year.............................................. . 1. ______2. Alimony paid in 3rd year................................................. 2. ______3. Floor............................................................................... 3. $15,000

51

4. Add lines 2 and 3............................................................. 4. ______ 5. Subtract line 4 from line 1............................................... 5. ______ (excess pay. for 2d post-sep. year—0 if neg.)6. Alimony paid in 1st year .................................................6. ______7. Adjusted alimony paid in 2nd year (line 1 less line 5).....7. ______8. Alimony paid in 3rd year ............................................... 8. ______9. Add lines 7 and 8 ............................................................ 9. ______10. Divide line 9 by 2 ........................................................ 10. ______11. Floor...............................................................................11. $15,00012. Add lines 10 and 11.......................................................12. ______13. Subtract line 12 from line 6.......................................... 13. ______ (excess payment for 1st post-separation year)14. Recaptured alimony. Add lines 5 and 13......................14. _______ If alimony was deducted, this amount must be reported as gross income in the third year. If alimony was received, this amount is deducted in the third year.

Child support obligations in default

Diez-Arguelles (Tax Court 1984) Facts: Husband becomes dead beat and stops paying child support. Wife wants to write off the bad debt he owes

her and deduct it See 166: Bad Debts

1. IRS says she can't take this as a non business bad debt2. 166 d says that are deductible in the year that they become completely worthless

1. Even if when you reach the point that you despair that husband will be a dead beat forever, there is no SOL on child support

2. So it will never become completely worthless because she has a legally enforceable right3. Instead court goes to 166 b

1. She has no basis and therefore she has no deduction Court says she wouldn't even get her foot in the door if she hadn't supported the children

1. And then the court says she has no basis 2. Tax court has never explained why they don't have basis 

1. No actual reasoning  Doesn't make them whole even if she did get it because its only a deduction Has not been overturned or modified in tax court So if you're her lawyer how do you do this?

Go back and challenge again...but need to make it non frivolous, i.e. with a law review article saying why it’s dumb

166(d): non-corporate taxpayer can deduct non-business bad debts as a short-term capital loss in the year such debts become completely worthless.

166(b): bad debts are deductible only to the extent of the tax-payer’s basis Here, court continues to deny the deduction since there is no basis due to the fact that she had not “out of pocket”

expenseo NB thinks this is just wrong and that it has been established that she (1) has basis and (2) the debt is

hopelessly lost and unrecoverable

Perry Deduction for bad debt again not allowable, court tells taxpayer to go to Congress and get it fixed there 1.166-1(c): to claim a deduction for a bad debt there has to be a valid an enforceable obligation to pay a fixed or

determinable sum of moneyo But, in Diez, the debt was valid, enforceable, and clearly could not be collected

When the law is bad like this, you should use persuasive commentary or get professors to write a note or amicus brief on the issue [especially in tax where you can go for a split in the law between tax court, court of claims, circuits]

HOW CAN TAXABLE INCOME AND TAXES BE REDUCED?

DEDUCTIONS (Itemized), EXEMPTIONS, AND CREDITS52

When considering income, after you determine how much and when, you want to see if you can whittle down your taxes due to the lowest amount possible Our rules on deductions, exemptions, and credits (DEC) work in such a way that people may have very

different zero brackets even if they have the same family sizeo It all depends on how they take advantage of the various DEC

Phaseoutso Phaseouts…Personalized exemptions and itemized deductions are reduced as adjusted gross income

rises above certain threshold amounts As income rises, the tax benefit of something incrementally declines

o Phase-outs amount to little more than back-door increases in the marginal tax rates. But, these hidden taxes are in no way large

o The Tax Code is being changed so as that phase-outs may no longer exist in the near future Personal deduction

o In addition to itemizing or claiming the standard deduction, all taxpayers are entitled to a personal exemption deduction for themselves and for each of their dependants.

Standard deduction v. itemized deductiono You can take either the SD or the ID, but not botho From here forward, we will be considering itemized deductions…

You only standard deduction if your itemized deductions do not exceed it Deductions operate like public subsidies

o We have to consider under what circumstances people in this country would be willing to take on a high tax burden…KEY RATIONALE

Topsy-turvy benefit of itemized deductionso Generally, Higher income higher marginal rates deduction is more valuable per $ regressive effect

Could change this, as WI has, by making deductions into credits so that they are the same dollar amount for everyone

Flow chart (p.333)Gross income (§61)

This is NOT an exhaustive list, no common theme Includes

Explicit exclusions Implicit inclusions

Minus Above the Line Deductions (§62a) Above the line v. below the line

If it is above the line, then you don’t lose it if you decide to take the standard deduction below the line

o If you are a lobbyist and you can move a deduction from below to above, you have made you clients a lot of money

But the value of the deduction may depend on your marginal tax rate  Above the line deductions work the same way as exclusions from income This is an exhaustive list

These are the 20 things, that’s it…no other deduction is an above the line deduction Examples mentioned by NB (§62a)

(a)(2)(D): certain expenses of elementary and secondary school teachers (a)(3): losses from sale or exchange of property (a)(7): retirement savings (a)(10): alimony (a)(15): moving expenses: 50 miles and changed employers (a)(17): interest on education loans

o If you make more than 50k you can’t deduct student loan interest, phase out provision

(a)(18): higher education expenseso tuition and fees

(a)(19): health savings account

53

Equals ---------------------------Adjusted Gross Income (§62) --------------------------------------------------------Minus Below the Line Deduction (EITHER standard ded. [§63(c)] OR itemized deductions [63(d)])

STANDARD DEDUCTION SD Amounts:$6,100/per person standard deduction; 12,200/married jointly; 8,950 head of

household; Chose between standard deduction and itemized--whichever is bigger

ITEMIZED DEDUCTION Not an exhaustive list:

o gifts to charityo state and local taxeso medical expenseso interests paid on home mortgages o casualty and theft losseso job expenses

***Below the line deductions are not listed in one place In essence, every other deduction

Notice that for some itemized deductions that there are thresholds…always consider whether the SD would be greater than the total amount you could deduct for itemizing

And Deduction for Personal Exemptions (§151) PE Amount = $3,900 per person in 2013 So zero bracket is at least equal to 10,000  Zero bracket can be different for every person!

Many will have the same because they don't itemize Equals Taxable Income (§63)

Personal Exemptions: You get 2 personal exemptions for joint returns and for married separate where the spouse has no gross income and is not another's dependent 151 b

o  151 c provides an exemption for each dependent as qualified in 152 Must be either a qualifying child or relative 

CHILD: o is a child, a child's descendant, or a sibling of the TPo is less than 19 years old, or if a student, less than 24 years oldo has not provided more than half of his or her own support; ando has the same principal place of abode as the TP for more than one-half of the taxable

year RELATIVE

o Child or child's descendant, parent or parent's ancestor, sibling, aunt, uncle, cousin, or in-law, or has the same principle place of abode as the TP and is a member of the TP's household

o has a gross income less than the exemption amounto received more than half his or her support from the TP and o is not a qualifying child of the TP

For parents not filing a joint return, the exemption for a qualifying child goes to the parent with whom the child reside for the longest period of time, and in case of a tie, to the one with the higher AGI

Divorced or legally separated parents are allowed to assign exemption however they agree  SIMPLE EQUATION

o Gross income – ATL = AGI – [(greater of BTL OR standard deduction) + personal exemption] = Taxable Income

Exam Review HypoSingle person with 2 kids Personal exemption = 3,700 x 3(2012 number)

54

o 11,100 total Standard deduction

o Head of household So, SD = 8,500 (2012 number)

Itemized deductionso Charitable contributions = 2,000

2,000 deduction you can take up to 50% of your adjusted GI

o Unreimbursed employee expenses = 8,000 7,000 deduction

2% thresholdo Mortgage interest = 1,500

1,500 in fullo State and local taxes = 1,200

1,200 in fullo Medical expenses = 3,000

None 7.5% threshold

AGI = 50,000o Possibilities

(1) AGI – SD – PE = TI No itemizing (SD)

o 50,000 – (11,100 + 8,500) = 30, 400 TI Itemize if you can do better than 30,400

(2) AGI – ID (itemized deductions) – PE = TI Itemizing

o 50,000 – (11,100 + 12,000) = 26,900 TIo We go with Possibility (2)

DEDUCTIONS Below, we are dealing with all itemized, below-the-line deductions

o Aside from Section (1) Health Insurance, which is not even income

Health Insurance §162: deductible for employer §106: excludable from gross income for employees Health insurance is a completely nontaxable event

o The purest subsidy you can get through the Code Examples

o (1) E (employee) receives

70k in salary 5k in health insurance

HS income: 75k Because of 106…

5k is excluded and 70k is her gross incomeo (2)

S (self-employed): 75k in receipts 5k in health care expenses

o deductible under 162 GI: 70k

o (3) N (employed, but not provided with health insurance by employer):

55

75k in salaryo both E and N cost employer 75k a year

doesn’t matter for the business tax-wise because salary and health are both deductible for the employer

5k in health care expenseso This 5k is a huge assumption because if you are a non-covered employee looking

for health insurance, you will get nowhere near as close to the same deal GI: 75k

Medical expenses

Extraordinary Medical Expenses o Under 213 a, medical expenses are deductible to the extent that they exceed 10% of adjusted gross income

Threshold seems to reflect the same kind of effort to distinguish between extraordinary misfortunes and the ordinary vicissitudes of life

Also reduces both the number of TPs who need to figure out each year precisely how much they spent on medical care and the resources the service and the courts must devote to the process of verifying medical expense deductions 

Reduces your expenses by your marginal tax rate...doesn't mean American public picking up your entire bill

No double counting, can't be covered by health insuranceo Because deductibles incentivize people to be less wasteful, Congress in 2003 enacted a new tax benefit for

Health Savings Accounts The idea behind this program is that people with money in HSA accounts will be relatively careful in

incurring medical expenses because they will be spending their own money HSAs do cover all medical expenses though, usually much more broad than medical insurance--

spending tax free dollars on these things  Policy:

o Amounts spent for extraordinary medical expenses do not provide consumption in the ordinary sense and therefore are simply not part of income

o Individuals who pay their own medical costs relieve the gov't of an expense that it would otherwise be obliged to bear

o The deduction is a proper encouragement to people to take good care of themselves, it is a useful subsidy for medical care

o In many instances, an injury or illness stems from work or interferes with ability to work so the cost of medical care should be regarded as cost of producing income 

Introduction What about medical expenses that are NOT REIMBURSED BY ANY MEDICAL COVERAGE you may have?

§262 Deductions cannot be personal, living, or family expenses except as otherwise permitted in the Code

§213 (a) allowance of deduction

o You can deduct medical expenses, not covered by insurance or otherwise, for yourself, spouse, or dependents to the extent that such expenses exceed 7.5 percent of adjusted gross income.

My aside: 213 kind of works like insurance…however, the funds after the “deductible” threshold are not covered, just merely not included in income

213 creates a disincentive to purchase health insuranceo If your AGI is 80k, 10% is 8k

So if you have 8.5k medical expenses beyond what health insurance covers BUT you can only deduct that which exceed 8k so 500!

(b) limitation with respect to medicine and drugs

56

o We are talking about prescription medicine, not Advil (d) definitions

o (1) “medical care” medical care is deductible a bit of broad definition

fleshed out below (1)(A): for the diagnosis, cure, mitigation, treatment, or prevention of disease, or for the purpose of

affecting any structure or function of the body (1)(B): for transportation primarily for and essential to medical care (1)(C): for qualified long-term care services, when provided by licensed health care practitioner

o (2) amounts paid for lodging lodging away from home, that is part of the medical care, is deductible you are not there for pleasure or spending money lavishly because you know it is deductible amount taken into account for each individual shall not exceed $50 per night

o (9) cosmetic surgery “medical care” does not include cosmetic surgery unless it is performed to cure a birth defect,

an injury stemming from an accident or trauma, or a disfiguring diseaseo Gender reassignment surgery?

Yes, but post re-assignment surgeries like breast implants are cosmetic and don't count  9th Circuit split the difference 

o Service has to be provided by a license provider Can't be a family member taking care of you, like a woman taking care of her parents

RR 75-318 – the cost of Braille books was deductible over the cost of regular books RR 64-173 – deduction allowed for cost of person to walk with a blind child through the halls Regs §1.213-1(e)(1)(iii): Outlays to make a house handicap accessible, or as necessitated by illness are only

deductible to the extent they do not increase the FMV of the houseo E.g., Constructing entrance/exit ramps; Widening doorways; Modifying hallways ; Installing railing, support

bars, or other modifications to the bathroom; Lowering or making adjustments to kitchen cabinets; Installing porch lifts or other forms of lifts; Modifying fire alarms, smoke detectors; Modifying stairs; Adding handrails or grab bars; Modifying hardware on doors; Modifying areas in front of entrance and exit doors; AND Grading of ground to provide access to residence.

The rules on what constitute “medical care” are very much socially contingento While falls under “cosmetic” and therefore a little too weird to constitute a deduction for “medical care” will

change over time as different procedures become more mainstream

Health Savings Accounts Allow you to saving money for health care 223(c)(2) – deductible must be at least $1K and not more than $5K An employee that has a health care plan within this can claim tax deductible contributions to the LESSER of (1) the

amount of the deductible or (b) $2250 (self coverage)/$4500 (family coverage)o Contribution to HSAs by employers are deductible to employers and nontaxable to employee recipients

These contributions are considered an adjustment to gross income under §62(a)(19) and t/f can be claimed by those who do NOT itemize

o Deduction is also not subject AMT §233(e) – Amount earned in the accounts are not taxable §223(f)(1) – Amounts paid from the account to reimburse employees for medical expenses are nontaxable to the

employee §223(f)(2) – Amounts paid out that are not for medical expenses are taxable as ordinary income In effect this serves as a retirement account as the deduction is allowed even though amounts were left over from prior

years coupled with the broad allowable use of this money (213 term is generally broader than insurance definition) gives people an incentive to careful in incurring medical expenses because they will be spending their own money

Further defining “medical care” Taylor (Tax Court 1987)

57

o Court held that TP could not deduct lawn mowing fees as a medical expense (it was a personal expense) Some factors to consider

(1) The expense is not necessarily directly tied to healthcare providers, etc. o Someone in the family could have helped

However, some things are medical expenses that don’t have anything to do with healthcare providers (consider lodging provisions)

(2) Causationo It is not clear whether “but for” his allergy he would have otherwise mowed his own

lawn Important to look to the past here

(3) Doctor recommended activities do not always constitute deductible medical expenseso Pointed to Altman case...guy with emphysema trying to get deduction for doc-recommended golf

Court basically says if we’re not giving emphysema guy a deduction hay fever guy isn't getting one o Gym memberships also not going to counto TP has the burden in these situations

Default is 262..rebuttable presumption against deduction

Henderson I (Tax Court Memo 2000-231)o Depreciation of the van used from disable kid will not be deductible as a medical expense because

deprecation does not constitute “amounts/expenses paid” The lift installed was deductible in itself

“Amounts/expenses paid”o When you pay an expense, money must change hands

Court is saying that “amounts paid” means dollars changing hands, but this isn’t really the case in an accounting sense (consider the change in HS income that the parents experience here)

o However, it reasonable to say that the depreciation constitutes an amount paid in that it lowers their HS income (NB thinks this case came out wrong)

Ochso Facts: During the taxable year, P was the husband of Helen and they had 2 kids

On Dec 10, a thyroidectomy was performed on P's wife which revealed cancer P sent kids to boarding school and deducted the cost of 1450 as a medical expense under 213 

Doctor told them to separate the children from the wife so her health could improve and she would stay in remission 

Kids stayed at boarding school until mom deemed recovered from cancer o Majority dismisses this as 262 personal expenses since someone would have needed to take care of kids even

if there was no sick mother – but this ignores the fact that there IS a sick mother Majority is worried about permitting a deduction for what they view as a personal expense that also

happened to result in a medical benefit to the wife Prof: THIS IS WRONG WAY TO ANALYZE

o Dissent acknowledges that the majority is worried about a slippery slope and so proposes his own test Would taxpayer normally spend their money in this way? No…didn’t got to private school before or

after illness Did the taxpayer buy such luxuries in the past? No Did a doctor prescribe this? Yes Did taxpayer act in most economically efficient way? We can assume they did, even if its deductible,

they don't have an incentive to throw away their money on something they can't afford with their budget

Was this over and above normal living expenses? Yes Was this closely geared to a particular condition rather than general health? Yes

o Dissent: Allow the deduction since Ochs’s wouldn’t otherwise have sent kids to boarding school but for mom’s cancer—deduction is for medical care, NOT for cost of hiring someone else to raise the kids/do the laundry etc…

PROF agrees with the dissent

58

Charitable Contributions

Introduction One of the safest bets in the world that big time tax reform efforts will never get rid of this Charitable contributions account for large sum of money, but it is not going anyway because it is considered

absolutely fundamental to American values Also, charitable contributions are highly abused

Two sides of the story for charitable contributions 170: governs deductibility of charitable contributions

o If you make a contribution to a charity, and you are itemizing, you can take the amount you give as a deduction

Charitable contributions are allowed as itemized deductions…above the lineo Any given year the maximum you can take is 50% of your adjusted gross income

But, 5 year carrover allowed Thus, another place, in addition to the medical expenses context (think the 7.5% threshold),

where adjusted gross income is importanto (b)(1)(B): if donation is principally for a private foundation it is limited to 30% of AGIo (b)(2): corporations are limited to donations of 10% of taxable incomeo (c): categories of charities

(1): governments (2): non-governmental charities

o (e)/(f): compliance provisions (f)(8): TPs who claim a deduction for any form of contribution in excess of $250 must be able to

substantiate the reduction with a written acknowledge of the donation by the donee. (f)(8)(D): the donee organization can alternatively file information with the IRS directly (f)(11): for contributions which are not readily valued and for which the deduction is more than $500,

the TP must include with the return a description of the property (f)(12): when vehicles are donated the deduction is limited to the proceeds of the sale

Charity has to give you a receipt that says that  For NB’s donation had to bluebook FMV because the charity didn't turn around and sell it-it

was a HS autobody class 501: governs the recipients of charitable contributions

o (c)(3): if you organized properly as a charity, the charitable contributions that you receive are not taxable

o Slight non-overlap -Buchanan doesn't care about this non overlap Just assume 501 c 3 charities can accept donations that the donor can deduct 

o Can't lobby, can't contribute to political campaign, can't inure to the benefit of any private SH or individual Hard to enforce the last part....very difficult where there are lots of followers and power

170 charities = 501 charitieso Requirements for “charity” status

(1) Charitable institutions cannot privately benefit individuals from the tax-exempt status. The private benefit is interpreted to mean “extraordinary benefit”

o There is a reasonable element to this (2) You cannot engage in electioneering

You cannot engage in candidate advocacyo But this doesn’t seem to be enforced so strictly.

Contributions of Property p 357o The difference between long-term and short-term

This is really important because the tax treatment on short-term is not very generous Short-term: If you have held the property for less than a year, the only thing you can deduct

is the basis

59

Long-term: If you have held the property for more than a year, you can deduct the FMV of the property.

Deductions for gifts of property that would be long term capital gains are limited to 30% of AGI (20% to a private foundation)

Policy o Way for the government to help subsidize charities

But, where do we draw the line when deciding what TPs are going to be subsidize?o Is this gov'ts way of supporting charitable gifts or does it just a windfall to those who would give anyway?o A decline in the marginal tax rates of donors reduces the value of the tax deduction 

If the tax deduction is an important factor in charitable giving, then donations should decline with falling marginal rates 

Donations would conversely rise as marginal rates rise, as the deduction becomes more valuable o Economists say that the dollar efficiency of the deduction is quite high 

Deductions stimulate donations and the dollar gain to the supported organizations equals or exceeds the dollar lose to the fisc. 

o Congress could establish a floor for deductions, i.e. only to the extent that they exceed 3% of TPs AGI  Could change the charitable contribution deduction!

Reduce value of the deduction to the TP that undoes the topsy turvy nature of this caused by progresive tax rates

o If you give 1k to charity and youre in the 40% tax bracket, it costs you 600o If you give 1k to charity and you're in the 10% tax bracket, it costs you 900

Prob not going anywhere

Gifts with private objective or benefits: BUSINESSES Ottawa Silica (Fed. Cir. 1983)

o Facts: TP acquired a bunch of ranch properties in Oceanside  Oceanside asked the TP if it would donate 50 acres of its land for a HS, and it was clear to TP that

they would be required to build access rods to the HS which would benefit the TP  TP eventually decided to donate plus an extra 20 acres for right of way for two access road and

claimed a deduction of 415k for the value of the property contributed o OS loses deduction for charitable contribution because of the roads they expected to be built in returno Substantial benefit test

You will NOT be allowed to deduct if you receive a substantial benefit in return for your charitable contribution, regardless of whether the predominant purpose of the contribution was charitable abd not business-related

“substantial benefit”o Something beyond what everyone else from the supposed act of charity

It is okay for a donor to get a benefit from a nominally charitable deduction so long as long as the benefit is not uniquely beneficial to the donor

Contra e.g., you can get a deduction for donating to an anti-gang organization

When do we know its a QPQ? Its okay if EVERYONE benefits, including the person who makes the donation  This is distinguished from things that substantially benefit the donor

o Substantial doesn't necessarily mean big it means of substance  So if you get a benefit that is of substance to you and to the public at large,

then this is not a deductable donation  So much information in the record that shows that they did this for

themselves How much does it matter that Ottawa planned it in advance? Even anticipating receiving it is enough! even if you don't ever get

the  benefito If this is really a business transaction in disguise than a deduction is not allowed o Very TP unfriendly

60

o What happens when OS loses its charitable donation? Although OS loses the charitable donation, they get to take advantage of the basis

The basis in the land that they gave away can now be added to their basis.o Had the taxpayer won, it would have recognized a current deduction equal to the

FMV of the contributed property. Instead, the TP received no current deduction and could only add to the basis of the contributed property to its other land. The TP would, in effect, be able to deduct that basis when and if it sold its other land.

The effect of the government victory was twofold. First, the taxpayer was forced to defer any tax benefit for what might turn out to be many year. Second, the eventual tax benefit would be limited to the basis of the contributed property, rather than the fair market value of the contributed property

Singer case o Singer wants students to buy their sewing machine in the future! That's valuable to them

Duval (Tax Court)o Tax Court went other direction from OSo Predominant purpose test (predominant purpose of the donation must be for a public purpose)

(1) Intent-based (2) More than incidental

o Very TP friendly For the IRS to win a case under this test, they have to prove that the TP knew they were going

to get a benefit AND that’s the biggest reason why they engaged in the transaction. “biggest” v. “substantial”

Law from OS and Duvalo There are at least two different standards out there

(1) substantial benefit (2) predominant purpose

o There is a possibility that a business may lose its charitable deduction for engaging in a quid pro quo The two different standards explore how much self-dealing is allowed

o Businesses who take charitable deductions need to be able to prove that they did not intend to receive a substantial benefit in exchange

o All or nothing test If you fail the standard, then you lose the entire value of the charitable deduction

All you get to do is take advantage of the basis of the property given away This rule seems very harsh… (next section)

o Important aside: The deduction itself cannot be the economic benefit that disallows the charitable deduction …you need to get something beyond the tax benefit

Gifts with private objective or benefits: NON-BUSINESSES

The same two contrasting standard may apply: o (1) substantial benefito (2) predominant purpose

Important distinction between business and non-businesso If a business makes a charitable contribution and gets too much benefit, then they lose the whole thingo RR 67-246: For non-businesses, there can be a quid pro quo

The existence of the quid pro quo means you lose only the value of the thing you receive as a part of that deal

This does not exist for businesses because, as a policy matter we strongly suspect businesses as seeks profits…so we need a relatively severe regime

An OS-like rule would kill much of charitable giving, so instead we have a partial reduction rule

61

It has long been the rule that the amount of any non-business deduction for a charitable contribution is limited to the excess of the payment to the charity over the value of any benefit received by the donor 

It is the value to the donor that counts: assumed to be the FMV of whatever is received by the donor, i.e. what it would cost donor to purchase the item or service

Non-businesses o Charity gets full advantageo Donor gets partial deduction

The deduction you get comes from the amount of your donation minus the value of the goods you received in exchange

The deduction is offset by the substantial benefit you receive in returno De minimis exception

You don’t have to offset your charitable deduction when the only thing you receive is very minimal (e.g., swag).

One of the problems is substantiating valueso 6115 polices this by requiring that for any quid pro quo contribution over $75 the charity must provide the

donor with a written statement that the entire amount is not deductible and must provide a “good faith estimate of the value of the goods or services” received.

Items below the $75 threshold are still not entirely deductible, there is simply no rule requiring charities to provide documentation

o TPs who claim a deduction for any form of a contribution in excess of $250 must be able to substantiate the deduction with a written acknowledgment of the donation by the donee organization 

Must state FMV of the goods or services 170 f 8 Cancelled check not sufficient  Or donee organization can file with the IRS 170 f 8 D 

o 170 f 11: For contributions of property that is not readily valued and for which the deduction is more than 500, the TP must include with the return a description of such property and such other information as the Secretary of the Treasury may require 

Although there are huge psychic benefits to charitable donations, the IRS does not offset the psychic benefit you get from the amount of the charitable deduction

Religious Benefits and Serviceso Can deduct regular contributions to the church and high holy day attendance 

But not for attending religious schoolso Hernandez case

SC upheld the Commissioner's disallowance of a deduction for amounts paid by members of the Church of Scientology for individual training and auditing 

IRS said that they were not contributions, they were QPQ for an identifiable benefit: auditing and training sessions 

Voluntarinesso Lombardo case

Because of felony charges, D was required to pay 145k to the county school fund which he claimed charitable deductions for

Court denied deductions because he made the payments in order to avoid going to prison so clearly not a gift

Private charitable contributions in short: Business…all or nothing rule

Non-business…quid pro quo reduction rule

The special case of collegiate athletics 170(I): allows for a deduction of 80% when contribution would be deductible but for the direct/indirect option to buy

ticketso NB says this is an example of law taking a backseat to politics

What is charitable?62

Bob Jones Facts: University has interracial students but they can't date each other

o IRS says because of this policy you are not a charitable organization Even though there’s nothing about this in the IRC, the charitable deduction is meant to embody the

common law understanding that in order to qualify for the charitable deduction the charity cannot violate public policy 

IRS promulgates a revenue ruling that has a non discrimination requirement for university 501 c 3s: says you can't have race based rules

o Bob Jones challenges this in court To qualify for tax exempt status an organization must (1) fall into one of the enumerated (8) categories AND (2)

promote a general public policy of charity Revocation of charity status on PUBLIC POLICY GROUNDS

o Really heavy burden There has to be “no doubt” that the charity violates a “fundamental public policy”

In short, there is a public policy exception, but the reading of “public policy” is so narrow that the exception is essentially limited to the fact of BJU (race-based discrimination).

o We have to consider what the average TP would be willing to “subsidize”

501(i): disallows charitable deductions for racially discriminatory social clubs, let alone educational institutionso does not disallow charitable deductions for gender-biased social clubs

fraternal organizations wouldn’t have it TAKEAWAY:

o Only outright racism in education will violate public policy  Powell concurrence, basically limits Bob Jones to its facts

o Never been applied since-almosto Overly sexist doesn't violate fundamental public policy (see 501 i --sex is not in there)

TAX CREDITS

Deductions v. creditso The basic difference is that the value of a deduction to a TP varies depending on what tax bracket they are in

A deduction only reduces your tax income by your marginal tax rate Tax deduction is never worth more than 35 cents on the dollar

A credit is always worth a dollar You get to reduce your tax owed by that amount

Difference is further complicated by refundable and non-refundable credits Also, deductions are non-refundable

Deductions are used to reduce your taxable income; however, credits are actually used to reduce the tax itself.

o Order that you would lobby Congress: Refundable credit

Most taxpayer friendly thing we have Non-refundable credit Above-the-line deduction Below-the-line deduction subject to no threshold Below-the-line deduction subject to some threshold Timing rule

Two types of creditso Refundable

You get the surplus of the credit minus the tax liability that you have if that number is positive.o Non-refundable

You tax liability goes down to zero, but you do not get the surplus if there is one. Why do people have zero federal tax liability?

o You are poor

63

o You have attained the amount of credits that has completely offset your tax liability.

§32: Earned income tax credit (EITC) Refundable tax credit, does not just offset liability but also results in payments (refundable credit)

o Under 32, a TP is entitled to claim a credit equal to a specified percentage of earned income up to a certain level 

Depends on the number of children, no children you get a small credit (max 475 in 2012) three+ children you get $5891

(k):o if you do anything wrong, you will lose the ability to attain future benefits

this provision is to stop cheating---harsh penalty, these populations don’t know the law! The EITC is there for the working poor with kids

Sometimes referred to as a wage subsidy or a negative income tax Phase-out

o At a certain threshold, you begin to lose benefits as your income increases You can go through an income range where you are worse off for earning more

This creates a tax disincentive to work and earn more Downslope=implicit tax

o As I earn more, my income doesn't go up dollar for dollar I get a dollar but i lose a fraction EITC

o Can become very high-happens with a lot of entitlement programso "Cliff effect"

Disincentive for earning more The EITC is biggest tax credit regime that we have

Tax credits, for the TP are…o Better than a deductiono Better when refundable

MIXED BUSINESS/PERSONAL DEDUCTIONSIntroduction -

§162 Allows deduction for “ordinary and necessary business expenses” paid or incurred during the taxable year in

carrying on any business or tradeo Doesn't have to be an incorporated business

Businesses don't have above or below the line, so if you are a business use 162 to deduct all expenses  For individuals, limited by 67

o 67: 2% of AGI threshold For this 2% threshold you can aggregate all of your miscellaneous “ordinary and necessary business

expenses”…..KEY: AGGREGATE 2% THRESHOLD FOR “ORGINARY AND NECESSARY BUSINESS EXPENSES”

Purpose of the threshold is to avoid dealing with de minimis expenses For businesses, there is no limiting threshold

o If self-employed, no threshold It’s not really “income” if paid out of pocket for production Below-the-line deductions

o Dealing with AGI threshold

§212 Applies to individuals For “cost of producing income” 212 was added specifically for income from sources other than a trade or business

o The expenses that go along with managing an investment portfolio

64

212 is probably unnecessaryo 162 is the go to section in terms of business deductions

Below-the-line deductions AMT: deduction under 212 of expenses of earning income are not allowed 56 b 1 A i

§262 No deduction allowed for personal, living, or family expenses

o We will not subsidize people’s personal expensesControlling the abuse of business deductions

Hobby losses

Nickerson (7th Cir. 1983) Facts: TP bought a farm as a desirable means of generating income, rented it out, fixed it up but didn’t expect to

make a profit for 10 years—in the meantime he deducted his losses Must be sincere, though not necessarily realistic expectation of profit

o Doesn’t have to be a REASONABLE expectation §183: activities not engaged in for profit

o Deduction allowed only when the activity is engaged in for profit (but only to the extent that it exceeds 2% AGI threshold)

Determined by a facts-and-circumstances test All-or-nothing approach…you fail 183, you get no deduction

o (b)(2): expenses incurred in activities not engaged in for a profit can be deducted only to the extent that the gross income derived from such activity exceeds otherwise allowable deductions (d): profit presumption (rebuttable)

If you made a profit in 3 of the last 5 taxable years, then you are presumed to have engaged in the activity for profit

If the TP does not meet the requirements for this rebuttable presumption, he has the burden of proof, and must show a bona fide expectation that at some point you will be making a profit

o 1.183-2(b): totality of the circumstances test Some factors considered:

Manner in which taxpayer carries on activity Expertise of taxpayer or his advisors Time & Effort expended by taxpayer in carrying on the activity Expectation that assets used in activity may appreciate in value Success of the taxpayer in carrying on other similar or dissimilar activities Taxpayer’s history of income or losses with respect to the activity Amount of occasional profits, if any, which are earned Financial status of taxpayer Elements of personal pleasure or recreation

o Policy Again, we have to ask when we are willing to see the activity as business focused enough so as to

provide a “public subsidy” via a tax deduction? Holding

o TP’s activity on the farm WAS for profit Court concludes that “[c]ommon sense indicates to us that rational people do not perform hard

manual labor for no reason, and if the possibility that petitioners performed these labors for pleasure is eliminated the only remaining motivation is profit.”

NB thinks that court comes out wrong in reversing the trial court on this rationale

Other hobby cases McCarthy: father was allowed to deduct managing costs of 13 year old son’s motorcross racing career even though

he had no likelihood of making a profit in any particular year

65

o Pre-opening expense doctrine under which expenses incurred before taxpayer begins business operations must be capitalized [unless they are a farmer]

Farish: Likelihood that taxpayer engaged in business to make a profit can be considered in light of geographical and occupational factors – breeding horses was found deductible for Texans engaged in oil drilling

Daily: Trips to Europe are not deductible as a business expense when antique dealers never advertised any items for sale, never offered anything for sale, and never actually sold an item

Home offices

Introduction Where a person does in fact use part of his home exclusively, or even primarily, for business the costs of that part of

the home (including a pro rate share of utility bills and deprecation on that part) might properly be regarded as a deductible business expense 

Congress intervened with home office fraud with 280Ao Begins with a general rule denying deductions for any use of a home for business purposes and then to list

specific concrete exceptions 

Popov (9th Cir. 2001) Facts: Popov is a professional violinist 

o Never given a place to practice from her employerso Has a one bedroom apt, used the living room as a home office 

Only used for practice 4-5 hours a day  Popov claimed a home office deduction for their living room and deducted 40% of their annual rent

and 20% of annual electricity bill  §280A: disallowance of certain expenses in connection with business use of home, rental of vacation homes, etc.

o (a) General rule is to deny deductions for any use of a home for business purposes technically unnecessary with 262 but serves as a reminder that personal expenses are non-deductible

o (c)(1): Subsection (a) shall not apply to any item to the extent such item is allocable to a portion of the dwelling unit which is exclusively used on a regular basis

(a) As the principle place of business for any trade or business of the taxpayer (b) As the place of business which is used by patients, clients, or customers in meeting or dealing

with the taxpayer in the normal course of his trade or business, OR (c) in the case of a separate structure which is not attached to the dwelling unit, in connection with

the taxpayer’s trade or business (if you set up a separate structure, the Code is more forgiving, and does require that business be principle use)

...exceptions get gradually more lenient ***280A(c)(1) is at issue here (whether professional musician is entitled to deduct expenses for the portion of her

home used exclusively for musical practice)o Court addresses whether this deduction is allowed on the grounds that it is a home office that is

EXCLUSIVELY used as the PRINCIPAL PLACE OF BUSISNESS for any trade or business of the TP Solomon test FOR DETERMINING PRINCIPLE PLACE OF BUSINESS

(1) Relative importance of the activities performed at each business locationo Point of performance should not determine the principal place of business o This prong is not too important to the court here

(2) Time spent at each placeo Court knows this is the money factor

Does the person spend significantly more time here than anywhere else where business activities occur?

This is where Popov wins—she spends more time here that at recording studios

o SO FIRST, EXCLUSIVE USE? THEN APPLY SOLOMON TO DETERMINE PPOB Take away

o Advise your client to engage in exclusive use There may be not that much left of (c)(1)(a) once your satisfy the exclusive use case

66

Office decoration

Henderson II (Tax Court) Facts: P was employed as an assistant attorney general by the state of SC

o Provided an office with furniture and furnishings o Bought a framed print for $35 and a life plant for $35 to decorate her office, and a $180 parking spot for rent;

wanted to deduct them all Occasionally used her own vehicle for business purposes when a State auto was not available

Where 162 and 262 may apply, 262 takes priority o Court finds that plants and prints in Henderson’s office were not deductible

Ruleo There essential inquiry is whether there is a “sufficient nexus” between the expense and the “carrying on” of

the individual’s trade or business to qualify the expenses for the deduction under 162(a), or whether they were in essence personal or living expense and non-deductible by virtue of 262.

To receive the deduction, the expense cannot be too “tangential,” “remote,” or “incidental” in aiding in the performance of the taxpayer’s business or duties

Her employer provided here with all the furnishings necessary to do her job 

Questions after the case:o Can’t deduct carpet while she’s there because that’s a private benefit

It would have to be once she leaves---just doesn't want to move it and says she’s donating it BUT not if she wants to leave it for her replacement that she knows

You’re allowed to help one person but you can't chose Deduction=FMV on the day she leaves

Aside: commuting expenses are nondeductible (somewhat addressing the parking issue that the court seems to ignore)

o Commuting expenses are personal

Automobiles, Computers, and other listed Property o 280f provides special rules for listed property

Computers Automobiles Cells Property generally of the type used for purposes of entertainment, recreation, or amusement Or anything else specified by regs

o Where listed property is used 50% or less for business purposes, depreciation is limited to straight-line under the normal useful life (as opposed to the shorter life available under ACRS)

o Where an employee acquires and uses listed property the business use requirement of this provision is not met except by use that is for the convenience of the employer and required as a condition of employment 280 f d 3

Some courts have held the condition requirement to be objective

Travel and entertainment expenses

Introduction:o If a person must go to another city on business, the cost of getting there and back is deductible o If they stay overnight, cost of food and lodging is also deductible o But what if there is substantial personal benefit from the trip?

Rule: deductible if the primary purpose is business Alternative test: was there a sufficient business justification for the trip?

If your employer pays for the trip you are not taxed on the value of the trip or your personal benefit received  o section 132do Still raises the issue though: whether the outlay is properly characterized as business related 

67

Employer provided outlays that would have been deductible if paid for directly by the employee do not constitute income to employee

Outlays that would not have been deductible if paid for by the employee are treated as taxable salary

§132 Employees are not taxed on whatever personal benefit they receive out of employer-provided business travel

o If the trip is characterized as non-business then it is treated as a form of salary and taxable incomeo The cost of employee business travel is deductible for employerso Non-business-related trip that an employer pays for is deductible to employer (along the same lines of

salary IF it meets ordinary and necessary business expense test) but is taxable to the employee

§274: In 1962, Congress enacted 274 which superimposes on the basic requirements of 162 additional rules for travel and

entertainmento 274 a 1 allows TPs to deduct their cost of any activity which is of a type generally considered to constitute

entertainment, amusement, or recreation, only if the activity if directly related to business  o But deductions are allowed for expenses that are associated with business and directly proceed or follow and

substantial and bona fide business discussion  Could stretch to a TP's entire life if he believes its all business 

Fifty percent limitation on meal and entertainment deduction o Section 274n limits the otherwise allowable deduction for meals and entertainment to 50% of cost 

EXPENSE OF SPOUSE O Where a person is on a legitimate tax-deductible business trip, no deduction is allowed for the additional

travel expenses of the person's spouse, unless the spouse is an employee of the person claiming the deduction, the spouse has a bona fide business purpose for going on the trip AND the additional expenses would otherwise be deductible 274 m 3

So can't just come along to entertain clients Section 274's substantiation requirements

O 274d: no deduction may be taken for traveling expenses, entertainment, or business gifts unless the TP substantiates by adequate records or by sufficient evidence corroborating the TP's own statement the amount, the time, and place of the event or the date or description of the gift, the business purpose, and the business relationship to the person entertained or the done 

Only that above $75

Rudolph Facts : An insurance co provided a trip from Dallas to NYC for group of its agents and their wives, including P

o Got the trip for selling a certain amount of insurance and got to bring his wife with him o Only had one business meeting a group luncheon, rest of the week was for pleasure o IRS assessed its value to them as taxable income 

No holding. US SC realizes that they should not have granted cert.o Found that the trip was just a different form of compensation/income (61) and the employee cannot

deduct it because it is not a business expense (162) as there was no compulsion to attend the trip Even though the employee couldn’t deduct here, the employer can deduct the expense as if he were

deducting salary TEST:

o Dominant motive and purpose test Is it related primarily to business or is it personal/for-pleasure in nature?

If for pleasure, the expense cannot be for business, and thus is not deductible.o Basically 162 v. 262

1.162-2(c): Where a taxpayer’s wide accompanies him on a business trip, expenses attributable to her travel are not deductible unless it can be shows that the wife’s presence on the trip has a bona fide business purpose. Incidental service does not qualify as a deductible business expense.

68

Dissent says every other professional who takes these kinds of trips gets  a deduction TAKE AWAY:

o When can you make a deduction in situations involving some degree of compulsion, but also entailing something that an objective observer may find to be enjoyable?

Very fact-specific inquiry

Planning guidanceo If you’re going on a trip, make sure that there is more business than pleasure going on

Make sure you have a written-down plan

1.162-2: Traveling expenses The traveling expenses have an all-or-nothing rule Even if we determine that the overall purpose of the trip was personal, you can still take deductions for particular

items on the trip that were business-relatedo However, you do not get to deduct a portion of your travel expenses

You can only deduct the travel expenses when you can deduct all of them because the trip was found to be business-related

Thus, the all-or-nothing nature of traveling expenses rule

Clothing expenses

Pevsner (5th Cir. 1980) P tries to deduct clothing as an “ordinary and necessary expenses” Facts:

o TP was a manager of a YSL boutique and is expected to wear YSL clothes while she works o Also wears YSL commuting to work, to fashion shows, and to business luncheonso TP did not wear the clothes during off work hours because they were too expensive for her simple everyday

life  Also didn't want to ruin them Could have worn them out of work though 

The generally accepted rule governing the deductibility of clothing expenses is that the cost of clothing is deductible as a business expense only if: (THREE PRONGS)

o (1) the clothing is of a type specifically required as a condition of employment (answer here: yes);o (2) it is not adaptable to general usage as ordinary clothing (answer here: no); AND

Objective test The clothing cannot be considered generally accepted for ordinary streetwear

o While this analysis isn’t completely clear, it is more clear than a subjective analysiso Also, this analysis is more horizontally equitable than a subjective analysis

Tough argument to make here because horizontal and vertical equity analyses seem both cut against one another and also collapse on themselves.

Horizontal equity -- could be argued either way, that two managers who both buy clothes should either both be allowed deduction or neither should be allowed, BUT in other direction this doesn’t work because if managers are of different socio-economic classes then it’s not horizontal

Vertical equity -- lower classes will be less likely to take advantage of deduction because less likely to have specialized uniforms [seems violative of vertical equity]

o (3) it is not so worn (answer here: yes)

Nelson- (Ozzie & Harriet CAN deduct clothes) They focus on situation specificness

o Which the Pevsner court seems to try to avoid

69

The deduction was allowed because the even though clothing worn by the cast members was support to exemplify Middle America, the show was filmed in Southern California

o i.e., cast members were not going to wear cardigans in Southern California

Mella case TP was a tennis professional  Claimed deductions for tennis clothes and shoes, which only lasted 2-3 weeks Court denied deductions, people wear those clothes all the time

Williams case TP rode a motorcycle in his business as an Amway distributor  He was allowed to deduct the cost of his leather uniform which had an Amway label on it

o But not the cost of the helmet or boots that he could use for non-business use 

Hierarchy of Tax Reduction Techniques (most TP friendly to least) Getting a credit (most are 100%) Expense your costs (entire deduction in current year) Depreciate Add to basis  Non recoverable (can't deduct credit nothing---full cost to you)

ALTERNATIVE MINIMUM TAX

Introduction o Imposes a tax at a reduced rate on a broader base (taking out things people ordinarily use to lower their taxes)

TP pays tax on the greater of the AMT or his tax liability as determined under normal taxo AMT was designed to ensure that high-income TPs paid at least some tax, however they organized their

financial affairs o Everyone agrees the AMT is a terrible provision 

BUT raises too much money for Congress to eliminate it  AMT is defined as the amount over what taxpayer would otherwise owe, so it is not one or the other, but rather

both, since AMT is defined as the excess We are only looking at Sections 55 and 56 and the individual (non-corporate) AMT The basic idea for the AMT is kind of like a flat tax

o The AMT is really close to a single rate tax Two rates

o 26% (up to 175k) and 28% (over 175k) To the extent the AMT is imposed based on timing, the payment is treated as a credit and can be used in later

years to reduce regular tax Structure of the tax

o Really big zero bracket Stands in for preferences eliminated by AMT

55: AMT (a): AMT is an add-on

o AMT = TMT (tentative minimum tax) – Regular tax This is your AMT only if you get a positive AMT You have no AMT liability if you get a negative AMT

o Total Tax = Regular Tax + AMT (b)

o TMTo Rate structure essentially flat

26% below 175k 28% above 175k

(d)

70

o Zero brackets for AMT, indexed to Rev Proc Phases out 

Once you reach a certain income, exemption begins to fade out 25 cents of the dollar Higher the income the smaller the zero bracket Also indexed in rev proc

Equation: AMT=Tenative minimum tax -regular tax (if positive) 55 ao What's the tentative min tax?

tax at rates set by section 55 applied to the taxable excesso What's the taxable excess?

TE=Alternative minimum taxable income-exemption amounto What is alternative min taxable income?

taxable income +adjustments in 56-58o **Everyone has to determine their TMT 

Parallel additional tax computation  IRS website does a safe harbor computation for determining when you have  to pay AMT....so you

don't need to figure out TMT  Big picture

o The difference between the two tax systems The AMT eliminates a bunch of deductions, credits, and special accounting methods

The question is which one’s get eliminated? … p.545o (6) Itemized deductions

Note that you still get your deduction for your home mortgage interest This is the biggest area where the AMT cuts back on things that normal

people take as deductions You lose the state and local taxes deduction You lose the deduction for interest on home equity loans You lost the deduction for certain job related outlays The deduction for medical expenses is limited to the excess over

10% (rather than 7.5%) of AGI You also lose the standard deduction and personal exemptions

The AMT has become problematic o There are rich people still getting their incomes low by using non-AMT preferenceso There are poor people suffering due to ATM-preferences o Roots of problems

AMT is not indexed to inflation Administrative problems

Hard for people to know whether they have to pay the AMT NB sometimes calls the AMT the “enough is enough tax”

o The government (and thus the citizenry) will stop subsidizing you at some point

Klaassen Facts: TPs were parents of ten dependent children and earned an AGI of 83k

o IRS found that they were liable for 1k AMT pursuant to 55-59  Required 3 specific increases to the taxable income

Requires the entire deduction for state and local taxes to be added back Reduced deduction allowable for medical expenses Eliminated deductions for themselves and children 

AMT Calculationo Normal/regular tax (1994) determination

AGI = 83,056 Minus Itemized deductions = 19,564

Medical expenses = 4,767 State and local taxes = 3263.56

71

Minus Personal exemptions = 29,400 12 personal exemptions: one for each of themselves and their 10 kids

Equals TI = 34,092 Regular tax = 5,111

o AMT determination (1) State and local taxes = 3263.56 0

56(b)(1)(A)(ii): All state and local taxes have to be added back under AMT (2) Medical expense deduction = 4,767 2076.41

56(b)(1): AMT reduces deduction for medical expenses from 7.5% AGI to 10% AGI threshold

o so, they lose 2.5% AGIo **NOW THAT’S GOING TO ALWAYS BE THE SAME YOU’LL LOSE ALL OF

THIS (3) Personal exemptions = 29, 400 0

55(b)(1)(E): in computing the AMT, 151 personal exemptions are not allowedo 55(d)(1): instead of 151 personal exemption, the AMT provides s substituted fixed

exemption for purposes of AMT computation AMTI = 68,832 Then, you get to take out the AMT exemption amount

Taxable Excess = 68, 832 (AMTI) – 45,000 (AMT exemption amount) = 23, 832 o 55(d): default and 2010/2011 AMT exemption amounts

(A): joint return/surviving spouse = 74,450 for 2011 (B): single (not a surviving spouse) = 48,450 for 2011

TMT = (26% and/or 28%) x 23,832 (Taxable Excess) = 6196 55(b)(i): determines whether to use 26% or 28% for individual TPs

o 26% of so much of the taxable excess as does not exceed 175ko 28% of so much of the taxable excess as exceeds 175k

AMT = 6196 (TMT) – 5111 (Regular Tax) = 1085 Positive AMT, so you have to pay it

o If negative, you don’t have to pay it Majority’s legal analysis in K

o Congressional intent Even if the TP can show legislative history pointing to the fact that this is a millionaires’ tax, what

Congress ended up doing is not at all what it intended to do The Code sections are very unambiguous and need to be taken at face value

o Constitutional arguments TP suggests that the tax infringes on their freedom to exercise their religion because it makes

additional children more expensive than they would otherwise be Court says that this statute is facially neutral, so tough luck

o For the TP to win, Congress would almost have to explicitly target the group being harmed

Court knocks this out of the park and says no constitutional problem at all Concurrence

o Pleads with Congress to solve the problems embedded in the AMT

Prosman (TC Memo 1999-87) 56(b)(1): in calculating AMTI, no deduction is allowed for miscellaneous itemized deductions and State and local

taxes, unless such amounts are deductible in determining AGI (i.e., above-the-line deductions) 56(b)(1)(E): there is no deduction for personal exemptions under 151 Also, even though the court sympathized with the lower income TPs at bar, the plain meaning of the AMT suggests

that they too are subject to it.o The AMT is not just for high income earners

Exam review Hypo

72

H and W filed join return in 2009 TI = 100k

o Regular tax = 17,375 AMTI = 150,000

o Taxable excess = 150,000 – 45,000 = 105,000 TMT = 26% of 105,000 (Taxable Excess) = 27,300 AMT = 27,300 (TMT) – 17,375 (Regular Tax) = 9,925

o Positive AMT, so you have to pay it Total tax = 27,300 (9,925 + 17,375)

The 2013 exemption amounts are:

73

Single taxpayers: $51,900Married taxpayers filing jointly: $80,800Married filing separately: $40,400Head of Household: $51,900